Sie sind auf Seite 1von 105

www.insightsonindia.

com
www.insightsias.com
TEST – 16 Solutions

Prelims 2016 – Test 16

Solutions

1. Consider the following statements about the All-India trends of


inflation.

Assertion (A): Urban inflation has always been more than rural Inflation
in post-independent India.

Reason (R): Urban India constitutes a larger share of India’s GDP than
rural areas.

In the context of the above, which of these is correct?

a) A is correct, and R is an appropriate explanation of A.


b) A is correct, but R is not an appropriate explanation of A.
c) A is correct, but R is incorrect.
d) A is incorrect, but R is correct.

Solution: d)

Justification: India's urban sector presently constitutes 63% of India's


GDP (up from 45% in 1990) and has been its key engine of growth
acceleration in the past decade. So, R is correct.

If you observe the trends given in India Yearbook for the year 2013
(Chapter 6) itself, the trends do not have a uniform pattern. It is more in
rural sometimes, and more in urban areas sometimes. A is incorrect.

Learning: With only 31% of India's population currently urbanized,


along with high population density and low GDP per capita, India's
urbanization trends have scope to significantly accelerate.

Q Source: Chapter 6: India Yearbook 2016

2. Remittances are an important source of financial flows for India.


Which of the following factors may lead to an increase in
remittance inflows to India?
1. Higher oil prices in the Gulf countries
www.insightsonindia.com
www.insightsias.com
TEST – 16 Solutions

2. Depreciation of Rupee

Which of the above is/are correct?

a) 1 only
b) 2 only
c) Both 1 and 2
d) None

Solution: c)

Justification: Statement 1: Higher oil prices help the Gulf countries


and other oil exporters to earn higher profits/income (where a large
number of Indian workers are employed – whose higher earnings made
higher transfers possible to India).So, 1 can be correct.

Statement 2: Depreciation of rupee increases the value of foreign


currency in India and hence the NRIs or OCIs are at an advantage
sending remittances to India.

Q Source: Chapter 10: Ramesh Singh: Indian Economy

3. Which of the following is/are the financial powers of the President


of India?
1. Money bills can be introduced in the Parliament only with his
prior recommendation.
2. He causes to be laid before the Parliament the Union Budget.
3. No demand for a grant can be made except on his
recommendation.
4. He can make advances out of the contingency fund of India to
meet any unforeseen expenditure.
5. He constitutes the Union finance commission to recommend
the distribution of revenues between the Centre and the states.

Select the correct answer using the codes below.

a) 3 and 5 only
b) 1, 2 and 5 only
c) 1, 2 and 4 only
d) 1, 2, 3, 4 and 5
www.insightsonindia.com
www.insightsias.com
TEST – 16 Solutions

Solution: d)

Justification: Statement 1 and 3: Both statements are correct. More on


these lines will be covered in the coming tests.

Statement 2: President shall, in respect of every financial year, cause to


be laid before Parliament, Annual Financial Statement as per Article 112
of the Constitution. The Budget is presented to Parliament on such date
as is fixed by the President.

Statement 4: The fund is held by the Finance Secretary (Department of


Economic Affairs) on behalf of the President of India and it can be
operated by executive action.

Q Source: Chapter 17: Indian Polity: M Laxmikanth

4. Ahmadabad Mill Strike 1918 led by Gandhiji was based on the


issue of
a) Plague Bonus of the previous year to workers
b) Inhuman treatment of mill workers by the management
c) Objection of management on participation of workers in the
freedom struggle
d) Large-scale layoffs of mill workers

Solution: a)

Learning: In 1918, there was a situation of conflict between the Gujarat


Mill owners and workers on the question of Plague Bonus of 1917.

The Mill Owners wanted to withdraw the bonus whole the workers
demanded a 50% wage hike. The Mill Owners were willing to give only
20% wage hike.

In March later, under the leadership of Gandhi, there was a strike in the
cotton mills. In this strike Gandhi used the weapon of Hunger strike.

It was carried out in pure non-violent disciplined way. The result was
that the strike was successful and the workers got a 35% wage increase.

Q Source: Chapter 12: Bipin Chandra: India’s Struggle for


Independence
www.insightsonindia.com
www.insightsias.com
TEST – 16 Solutions

5. Hannover Messe 2015 was a/an


a) Energy summit held under the aegis of Conference of Parties
(COP) of UNFCCC
b) The cultural exchange programme between India and Europe
organized by the Ministry of Culture and Ministry of Tourism
c) An annual global exhibition held in Germany which provided a
platform for India to gain entry to international markets
d) None of the above

Solution: c)

Learning: The Hannover Messe (HM) is one of the most important


annual global exhibitions which provided an ideal platform for India to
gain entry to international markets with focus on global technological
and industrial innovation held in Hanover, Germany in April, 2015.
‘Make in India’ was the theme of the Hanover Fair.

As India was designated as a “Partner Country” by Hanover Messe, a


rare distinction achieved by any country for the second time in the last
ten years, a high profile platform for the presentation of India’s
industrial and business achievements was geared up by this Department
in close coordination with the DIPP.

The high level political and economic attendance at HM brings


worldwide publicity.

6. When a vacancy occurs in the office of the President due to his


resignation, the Vice-President acts as the President until a new
President is elected. Consider the following with regard to when
the office of Vice-President is vacant.

Assertion (A): Speaker, Lok Sabha acts as the President of India in such a
case.

Reason (R): Speaker, Lok Sabha is ranked immediately below the Vice-
President and Prime Minister in the Table of Precedence.

In the context of the above, which of these is correct?


www.insightsonindia.com
www.insightsias.com
TEST – 16 Solutions

a) A is correct, and R is an appropriate explanation of A.


b) A is correct, but R is not an appropriate explanation of A.
c) A is incorrect, but R is correct.
d) Both A and R are incorrect.

Solution: d)

Justification: When a vacancy occurs in the office of the President due


to his resignation, removal, death or otherwise, the Vice-President acts
as the President until a new President is elected.

Further, when the sitting President is unable to discharge his functions


due to absence, illness or any other cause, the Vice-President discharges
his functions until the President resumes his office.

In case the office of Vice-President is vacant, the Chief Justice of India


(or if his office is also vacant, the senior-most judge of the Supreme
Court available) acts as the President or discharges the functions of the
President.

Q Source: Chapter 17: Indian Polity: M Laxmikanth

7. Which of the following statements about the Archaeological Survey


of India (ASI) is INCORRECT?
a) It was established before independence.
b) It functions as an attached office of the Department of Culture.
c) ‘Expeditions abroad’ is one of the major activities of the
organization.
d) ASI cannot take up conservation projects of sites outside India.

Solution: d)

Justification: The ASI has taken up the conservation project of Ta


Prohm, Cambodia under the ITEC programme of Ministry of External
Affairs with an outlay of ₹ 19.51 crore.

The conservation project has been started on the request of the Royal
Government of Cambodia for India’s assistance in Conservation and
Restoration of Prasat Ta Prohm.
www.insightsonindia.com
www.insightsias.com
TEST – 16 Solutions

The major activities of the Archaeological Survey of India are:

• Survey of archaeological remains and excavations;


• Maintenance and conservation of Centrally protected monuments,
sites and remains;
• Chemical preservation of monuments and antiquarian remains;
• Architectural survey of monuments;

Q Source: Chapter 5: India Yearbook 2016

8. In case of a Parliamentary deadlock, the President summons a


joint sitting of both houses to resolve the issue. Who chairs the
joint sitting?
a) President
b) Chairman, Rajya Sabha
c) Speaker, Lok Sabha
d) Leader of the House of People

Solution: c)

Learning: The President is an integral part of the Parliament of India,


and enjoys the following legislative powers:

• He can summon or prorogue the Parliament and dissolve the Lok


Sabha.
• He can address the Parliament at the commencement of the first
session after each general election and the first session of each
year.
• He can send messages to the Houses of Parliament, whether with
respect to a bill pending in the Parliament or otherwise.
• He can appoint any member of the Lok Sabha to preside over its
proceedings when the offices of both the Speaker and the Deputy
Speaker fall vacant.

Q Source: Chapter 17: Indian Polity: M Laxmikanth


www.insightsonindia.com
www.insightsias.com
TEST – 16 Solutions

9. Consider the following statements.

Assertion (A): All doubts and disputes in connection with election of the
President are inquired into and decided by the Election Commission of
India.

Reason (R): Election Commission of India is entrusted with the


responsibility of conducting the Presidential elections.

In the context of the above, which of these is correct?

a) A is correct, and R is an appropriate explanation of A.


b) A is correct, but R is not an appropriate explanation of A.
c) A is correct, but R is incorrect.
d) A is incorrect, but R is correct.

Solution: d)

Justification: Supreme Court decides all the doubts and disputes in


connection with election of the President even though ECI organizes and
conducts the elections.

So, A is incorrect, and R is correct.

Moreover, ECI has only limited quasi-judicial authority with regard to


elections of MPs or MLAs. We will be covering these topics in the coming
tests.

Q Source: Chapter 17: Indian Polity: M Laxmikanth

10. It is said that the US-India Defense Technology and


Partnership Act that proposes to amend the ‘Arms Export Control
Action’ is of immense strategic significance to India. What can be
the possible reason(s)?
1. It seeks to formalise India’s status as a major partner of equal
status as US’s NATO allies and closest partners.
2. It will elevate India's status in export of defence articles from
the United States to India.
www.insightsonindia.com
www.insightsias.com
TEST – 16 Solutions

Which of the above is/are correct?

a) 1 only
b) 2 only
c) Both 1 and 2
d) None

Solution: c)

Learning: Defence trade between the US and India is one of the


strongest areas of the bilateral economic relationship. Over the past 10
years it has risen from $300 million to over $14 billion.

The act has been introduced by Congressman George Holding, Co-Chair


of the House India Caucus. It will help

• Cement the India-US relations and lay a foundation for future


cooperation and growth.
• Shorten the time required for the notification of sale or export of
defence articles from the United States to India.
• Encourage more joint contingency planning and require the US
government review and assess India’s ability to execute military
operations of mutual interest.

Q Source:
http://economictimes.indiatimes.com/news/defence/resolution-
introduced-to-bring-india-on-par-with-nato-
allies/articleshow/51527876.cms

11. The Ghadar party was a revolutionary organization with the aim of
securing India's independence from British rule. Who among the
following were associated with it?
1. Lala Har Dayal
2. Rashbehari Bose
3. Sohan Singh Bhakna

Select the correct answer using the codes below.

a) 1 and 2 only
b) 2 and 3 only
www.insightsonindia.com
www.insightsias.com
TEST – 16 Solutions

c) 1 and 3 only
d) 1, 2 and 3

Solution: d)

Learning: The Ghadar Party was an organization founded by Punjabi


Indians in the United States and Canada with the aim of securing India's
independence from British rule.

Key members included Lala Har Dayal, Sohan Singh Bhakna, Abdul
Hafiz Mohamed Barakatullah, Kartar Singh Sarabha, and Rashbehari
Bose.

After the outbreak of World War I, Ghadar party members returned to


Punjab to agitate for rebellion alongside the Babbar Akali Movement.

In 1915 they conducted revolutionary activities in central Punjab and


organised uprisings. Their presence shook the hold of the British empire
and police surveillance in Punjabi villages increased in an attempt to
crush the rebellion.

Q Source: Chapter 12: Bipin Chandra: India’s Struggle for


Independence

12.Consider the following statements.


1. The Constitution does not contain any specific procedure for the
selection and appointment of the Prime Minister.
2. The Constitution does not require that a person must prove his
majority in the Lok Sabha before he is appointed as the Prime
Minister.

Which of the above is/are correct?

a) 1 only
b) 2 only
c) Both 1 and 2
d) None

Solution: c)
www.insightsonindia.com
www.insightsias.com
TEST – 16 Solutions

Justification: Statement 1: Article 75 says only that the Prime Minister


shall be appointed by the president. However, this does not imply that
the president is free to appoint any one as the Prime Minister.

In accordance with the conventions of the parliamentary system of


government, the President has to appoint the leader of the majority party
in the Lok Sabha as the Prime Minister.

Statement 2: In 1980, the Delhi High Court held that the Constitution
does not require that a person must prove his majority in the Lok Sabha
before he is appointed as the Prime Minister. The President may first
appoint him the Prime Minister and then ask him to prove his majority
in the Lok Sabha within a reasonable period.

Q Source: Chapter 19: Indian Polity: M Laxmikanth

13. In his oath of secrecy, the Prime Minister swears that


1. All government proceedings will be kept secret.
2. No matter handled by the Prime Minister’s Office will be
disclosed to other ministries.

Which of the above is/are correct?

a) 1 only
b) 2 only
c) Both 1 and 2
d) None

Solution: d)

Justification: The PM swears that he will not directly or indirectly


communicate or reveal to any person or persons any matter which shall
be brought under his consideration or shall become known to his as
prime minister for the Union except as may be required for the due
discharge of his duties as such Minister.

This does not imply that all government proceedings will be secret or
Pm’s office will work in confidentiality. So, both statements are
incorrect.

Q Source: Chapter 19: Indian Polity: M Laxmikanth


www.insightsonindia.com
www.insightsias.com
TEST – 16 Solutions

14.Consider the following about Earth Hour.


1. It is organized by the World Wide Fund for Nature (WWF).
2. The day organizes and encourages planting of at least one tree
by every human being at a specific time.
3. For India, the country-specific theme for 2016 was to encourage
usage of solar energy.

Select the correct answer using the codes below.

a) 1 and 2 only
b) 2 and 3 only
c) 1 and 3 only
d) 1, 2 and 3

Solution: c)

Learning: Earth Hour 2016 encouraged people to explore and adopt a


cleaner and more sustainable lifestyle by embracing renewable energy
solutions.

This day encourages citizens, communities, businesses and organisations


to switch the lights off from 8:30 pm local time for an hour to highlight
the plight of the planet. The first Earth Hour was held in 2007 in Sydney,
Australia.

Q Source:
http://www.thehindu.com/news/cities/Hyderabad/residents-look-the-
other-way-during-earth-hour/article8376534.ece

15. Exchange-Traded Fund (ETF) is


a) A basket of blue-chip company stocks
b) A basket of assets traded on stock market
c) FII that is received in India via the security markets
d) FDI in stock market companies operating in India
www.insightsonindia.com
www.insightsias.com
TEST – 16 Solutions

Solution: b)

Learning: The link explains the concepts very well

http://www.etf.com/etf-education-center/21011-what-is-an-etf.html

An exchange-traded fund (ETF) is an investment fund traded on stock


exchanges, much like stocks.

An ETF holds assets such as stocks, commodities, or bonds, and trades


close to its net asset value over the course of the trading day. Most ETFs
track an index, such as a stock index or bond index.

Q Source: Chapter 11: Ramesh Singh: Indian Economy

16.Consider the following statements.


1. The President may require the Council of Ministers to
reconsider advice tendered by it.
2. The advice tendered by Ministers to the President shall not be
inquired into in any court.

Which of the above is/are correct?

a) 1 only
b) 2 only
c) Both 1 and 2
d) None

Solution: c)

Justification: As per Article 74—Council of Ministers to aid and advise


President.

There shall be a Council of Ministers with the Prime Minister at the head
to aid and advise the President who shall, in the exercise of his functions,
act in accordance with such advice.

However, the President may require the Council of Ministers to


reconsider such advice and the President shall act in accordance with the
advice tendered after such reconsideration.
www.insightsonindia.com
www.insightsias.com
TEST – 16 Solutions

Q Source: Chapter 20: Indian Polity: M Laxmikanth

17. In 1971, the Supreme Court held that even after the dissolution of
the Lok Sabha, the council of ministers does not immediately cease
to hold office. How does it help the political system and
administration in India?
a) It prevents the bureaucracy from usurping political power.
b) It ensures the accountability of the executive to the legislature.
c) The Council of Ministers get an opportunity to again prove their
party majority on the house floor.
d) The president cannot exercise the executive power without the aid
and advice of the council of ministers, thus it helps him.

Solution: d)

Justification: Option (a): This option may seem correct as in absence


of council of Ministers, bureaucracy becomes an important decision-
making authority. But, such extreme generalizations should not be
chosen as the answer. (d) is more appropriate.

Option (b): Once the house is dissolved, the very idea of accountability to
the legislature dissolves for the time being. Only the President then
issues orders.

Option (c): Once the house is dissolved, the CoM cannot prove their
majority again. So, it is wrong.

Option (d): Article 74 is mandatory and, therefore, the president cannot


exercise the executive power without the aid and advise of the council of
ministers. Any exercise of executive power without the aid and advice
will be unconstitutional as being violative of Article 74’.

Q Source: Chapter 20: Indian Polity: M Laxmikanth


www.insightsonindia.com
www.insightsias.com
TEST – 16 Solutions

18. Consider the following about Cabinet Committees.


1. They are extra-constitutional bodies.
2. They are only temporary in nature.
3. Non-cabinet ministers cannot be its members.
4. Only the Prime Minister is authorized to chair cabinet
committees.

Select the correct answer using the codes below.

a) 1 and 2 only
b) 3 and 4 only
c) 1 only
d) 2 and 4 only

Solution: c)

Justification: Statement 1: They are not mentioned in the


Constitution. However, the Rules of Business provide for their
establishment

Statement 2: They are of two types—standing and ad hoc. The former


are of a permanent nature while the latter are of a temporary nature. So,
2 is wrong.

Statement 3: They usually include only Cabinet Ministers. However, the


non-cabinet Ministers are not debarred from their membership.

Statement 4: They are mostly headed by the Prime Minister. Sometimes


other Cabinet Ministers, particularly the Home Minister or the Finance
Minister, also acts as their Chairman. But, in case the Prime Minister is a
member of a committee, he invariably presides over it.

Learning: They are set up by the Prime Minister according to the


exigencies of the time and requirements of the situation.

They not only sort out issues and formulate proposals for the
consideration of the Cabinet, but also take decisions. However, the
Cabinet can review their decisions.

Q Source: Chapter 21: Indian Polity: M Laxmikanth


www.insightsonindia.com
www.insightsias.com
TEST – 16 Solutions

19.World Network of Biosphere Reserves includes which of the


following protected areas of India?
1. Gulf of Mannar
2. Sundarban
3. Namdapha
4. Kanchenjunga
5. Agasthyamala

Select the correct answer using the codes below.

a) 1, 2 and 5 only
b) 2, 3 and 4 only
c) 1, 3 and 5 only
d) 1, 2, 3, 4 and 5

Solution: a)

Learning: India’s Agasthyamala Biosphere Reserve (ABR) was included


in the World Network of Biosphere Reserves of United Nations
Educational, Scientific and Cultural Organization (UNESCO).

With the addition of the ABR, total 10 of the 18 biosphere reserves in the
country have made it to the list of World Network of Biosphere Reserves.

The other 9 Biosphere Reserves are: Nilgiri (Western Ghats), Gulf of


Mannar (Tamil Nadu), Sunderban (West Bengal), Nanda Devi
(Uttarakhand), Nokrek (Meghalaya), Pachmarh (Madhya Pradesh),
Similipal (Odisha), Achanakmar-Amarkantak (Madhya Pradesh and
Chhattisgarh) and Great Nicobar.

Q Source: http://www.thehindu.com/news/national/kerala/indias-
agasthyamala-among-20-unesco-world-biosphere-
reserves/article8381525.ece
www.insightsonindia.com
www.insightsias.com
TEST – 16 Solutions

20. Consider the following statements.

Assertion (A): Though the President of India is not a member of either


House of Parliament, he is an integral part of the institution of
Parliament.

Reason (R): A bill passed by both the Houses of Parliament cannot


become law without the President’s assent.

In the context of the above, which of these is correct?

a) A is correct, and R is an appropriate explanation of A.


b) A is correct, but R is not an appropriate explanation of A.
c) A is correct, but R is incorrect.
d) A is incorrect, but R is correct.

Solution: a)

Justification: He does not even sit in Parliamentary proceedings.

However, he also performs certain functions relating to the proceedings


of the Parliament, for example, he summons and prorogues both the
Houses, dissolves the Lok Sabha, addresses both the Houses, issues
ordinances when they are not in session, and so on.

Q Source: Chapter 22: Indian Polity: M Laxmikanth

21.The parliamentary form of government emphasises on the


a) Separation of powers between the legislature and executive
b) System of Universal Adult Franchise
c) Interdependence between the legislative and executive organs
d) Democratic election system administered by an independent body

Solution: c)

Justification: option (a) is valid in the Presidential system as in USA.

A Parliament may not be based on democratic elections or even adult


franchise. It can be an entirely nominated body as it largely used to be in
the colonial rule. So, (b) and (d) are incorrect.
www.insightsonindia.com
www.insightsias.com
TEST – 16 Solutions

In India, executive is a part of the legislature, and the legislature is led by


the executive. So, (c) is the most appropriate option.

Q Source: Chapter 22: Indian Polity: M Laxmikanth

22. Consider the following statements about Rajya Sabha.


1. The representatives of states in the Rajya Sabha are elected by
the elected members of state legislative assemblies.
2. The election to Rajya Sabha is held in accordance with the
system of proportional representation by means of the single
transferable vote.
3. The seats are allotted to the states in the Rajya Sabha on the
basis of population.

Select the correct answer using the codes below.

a) 1 and 2 only
b) 2 and 3 only
c) 1 and 3 only
d) 1, 2 and 3 only

Solution: d)

Learning: The seats are allotted to the states in the Rajya Sabha on the
basis of population. Hence, the number of representatives varies from
state to state.

For example, Uttar Pradesh has 31 members while Tripura has 1 member
only.

However, in USA, all states are given equal representation in the Senate
irrespective of their population.

USA has 50 states and the Senate has 100 members—2 from each state.

The representatives of each union territory in the Rajya Sabha are


indirectly elected by members of an electroral college specially
constituted for the purpose.
www.insightsonindia.com
www.insightsias.com
TEST – 16 Solutions

Q Source: Chapter 22: Indian Polity: M Laxmikanth

23. The Union Cabinet has given its approval for India to accede
to the Ashgabat Agreement. It establishes international transport
and transit corridor between
a) Central Asia and the Persian Gulf countries
b) South-east Asia to European Countries
c) West-African countries to South Asian countries
d) Oil producing Gulf countries and littoral states of Indian Ocean

Solution: a)

Learning: It is a transit agreement established in year 2011.

Its founding members are Uzbekistan, Iran, Turkmenistan and Oman.


Kazakhstan had joined the grouping in 2015.

Accession to the Agreement would enable India to:

• Utilise this existing transport and transit corridor to facilitate trade


and commercial interaction and ties with the Eurasian region.
• Synchronise India’s efforts to implement the International North
South Transport Corridor (INSTC) for enhanced connectivity.
• It will provide India an opportunity for reorientation of the freight
traffic from the traditional sea route to land transcontinental
routes.

Q Source: http://pib.nic.in/newsite/PrintRelease.aspx?relid=138309

24. The Constitution has empowered the Parliament to prescribe


the manner of choosing the representatives of the union territories
www.insightsonindia.com
www.insightsias.com
TEST – 16 Solutions

in the Lok Sabha. Members of Lok Sabha from the Union


Territories are
a) Chosen by direct election
b) Nominated by the Administrator/Lieutenant Governor of the
UT
c) Recommended by the Union Cabinet and chosen by the
President
d) Chosen by indirect elections by the local bodies of the UTs

Solution: a)

Learning: Accordingly, the Parliament has enacted the Union


Territories (Direct Election to the House of the People) Act, 1965, by
which the members of Lok Sabha from the union territories are also
chosen by direct election.

The representatives of each union territory in the Rajya Sabha are


indirectly elected by members of an electroral college specially
constituted for the purpose.

This election is also held in accordance with the system of proportional


representation by means of the single transferable vote.

Out of the seven union territories, only two (Delhi and Puducherry) have
representation in Rajya Sabha. The populations of other five union
territories are too small to have any representative in the Rajya Sabha.

Q Source: Chapter 22: Indian Polity: M Laxmikanth

25. Consider the following about the Delimitation Commission


of India.
1. It is a statutory body.
2. It redraws the boundaries of both assembly and Lok Sabha
constituencies.
3. Its orders cannot be challenged in a court of law.
4. The Lok Sabha cannot modify its orders.
www.insightsonindia.com
www.insightsias.com
TEST – 16 Solutions

5. It is setup every five years.

Select the correct answer using the codes below.

a) 1, 2 and 5 only
b) 3, 4 and 5 only
c) 1, 2, 3 and 4 only
d) 1, 4 and 5 only

Solution: c)

Justification: Statement 1: It is established by Government of India


under the provisions of the Delimitation Commission Act.

Statement 2: The main task of the commission is to redraw the


boundaries of the various assembly and Lok Sabha constituencies based
on a recent census.

The representation from each state is not changed during this exercise.
However, the numbers of SC and ST seats in a state are changed in
accordance with the census.

Statement 3 and 4: The Commission is a powerful body whose orders


cannot be challenged in a court of law. The orders are laid before the Lok
Sabha and the respective State Legislative Assemblies. However,
modifications are not permitted.

Statement 5: Delimitation commissions have been set up four times in


the past - In 1952, 1963, 1973 and 2002 under Delimitation Commission
acts of 1952, 1962, 1972 and 2002.

The present delimitation of parliamentary constituencies has been done


on the basis of 2001 census figures under the provisions of Delimitation
Act, 2002.

However, the Constitution of India was specifically amended in 2002 not


to have delimitation of constituencies till the first census after 2026.

Thus, the present Constituencies carved out on the basis of 2001 census
shall continue to be in operation till the first census after 2026.

Q Source: Chapter 22: Indian Polity: M Laxmikanth


www.insightsonindia.com
www.insightsias.com
TEST – 16 Solutions

26. Sevottam is related to


a) Citizen’s charter in public organizations
b) Grievance redressal programmes within the most vulnerable
communities
c) Reform of Public Sector Banks (PSBs)
d) Providing support to first generation entrepreneurs under
‘Make in India’ programmes

Solution: a)

Learning: The Department of Administrative Reforms & Public


Grievances (DARPG), Ministry of Personnel, Public Grievances and
Pensions has taken steps to put in place Sevottam Compliant Citizen’s
Charter and Grievance Redress Mechanism.

The Sevottam framework was designed by DARPG in 2006 as an


assessment improvement framework for public service delivery.

Q Source: Chapter 8: India Yearbook 2016

27. Consider the following about Rajya Sabha.


1. It is a permanent body and not subject to dissolution.
2. The retiring members of Rajya Sabha are eligible for re-election
and re-nomination any number of times.
3. The Constitution fixes the term of the members of the Rajya
Sabha.
4. The President can curtail the term of a Rajya Sabha member any
time based on the recommendation of the Union cabinet.

Select the correct answer using the codes below.


www.insightsonindia.com
www.insightsias.com
TEST – 16 Solutions

a) 2, 3 and 4 only
b) 1 and 4 only
c) 1 and 3 only
d) 1 and 2 only

Solution: d)

Justification: Statement 1: It is a continuing chamber, that is, it is a


permanent body and not subject to dissolution. However, one-third of its
members retire every second year.

Statement 2: The seats left are filled up by fresh elections and


presidential nominations at the beginning of every third year.

The retiring members are eligible for re-election and re-nomination any
number of times.

Statement 3: The Constitution has not fixed the term of office of


members of the Rajya Sabha and left it to the Parliament. Accordingly,
the Parliament in the Representation of the People Act (1951) provided
that the term of office of a member of the Rajya Sabha shall be six years.

Statement 4: The act also empowered the president of India to curtail the
term of members chosen in the first Rajya Sabha. But, he cannot do it
anytime based on any recommendations. The term of MPs is fixed.

In the first batch, it was decided by lottery as to who should retire.


Further, the act also authorised the President to make provisions to
govern the order of retirement of the members of the Rajya Sabha.

Q Source: Chapter 22: Indian Polity: M Laxmikanth

28. Which of the following is/are recognized as classical dance


forms in India?
1. Odissi
2. Manipuri
3. Sattriya
4. Mohiniyattam

Select the correct answer using the codes below.


www.insightsonindia.com
www.insightsias.com
TEST – 16 Solutions

a) 2, 3 and 4 only
b) 1, 2 and 3 only
c) 3 and 4 only
d) 1, 2, 3 and 4

Solution: d)

Learning: Dance in India has an unbroken tradition of over 2,000


years. Its themes are derived from mythology, legends and classical
literature, two main divisions being classical and folk.

• Classical dance forms are based on ancient dance discipline and


have rigid rules of presentation. Important among them are
Bharata Natyam, Kathakali, Kathak, Manipuri, Kuchipudi and
Odissi.

• Both classical and folk dances owe their present popularity to


institutions like Sangeet Natak Akademi and other training
institutes and cultural organizations.

• The Akademi gives financial assistance to cultural institutions and


awards fellowships to scholars, performers and teachers to
promote advanced study and training in rare forms of dance and
music.

Q Source: Chapter 5: India Yearbook 2016

29. The governor can reserve a bill passed by the State


Legislature for the consideration of the President in which of the
following cases?
1. If it is against the provisions of the Constitution.
2. If it is opposed to the Directive Principles of State Policy.
3. If it is against the larger interest of the country or of grave
national importance.
4. If it endangers the position of the State High Court

Select the correct answer using the codes below.


www.insightsonindia.com
www.insightsias.com
TEST – 16 Solutions

a) 1 and 4 only
b) 1, 2 and 3 only
c) 2, 3 and 4 only
d) 1, 2, 3 and 4

Solution: d)

Justification: There is one more case when the bill can be reserved
along with compulsory acquisition of property under Article 31A of the
Constitution.

In case the bill passed by the state legislature endangers the position of
the state high court, such reservation is obligatory.

Q Source: Chapter 26: Indian Polity: M Laxmikanth

30. The Speaker of the Lok Sabha derives his powers and duties
from the Constitution of India, the Rules of Procedure and
Conduct of Business of Lok Sabha, and Parliamentary
Conventions. Accordingly, she exercises which of the following
powers?
1. She is the final interpreter of the provisions of the Constitution
of India for any matter relating to the Parliament.
2. She has the final power in matters of decorum, order and
business in the house.
3. She decides whether a bill is a money bill or not.
4. She appoints the chairman of all the parliamentary committees
of the Lok Sabha and supervises their functioning.

Select the correct answer using the codes below.

a) 2 and 3 only
b) 2, 3 and 4 only
c) 1, 2 and 3 only
d) 1 and 4 only

Solution: b)

Justification: Statement 1: Two things should be noted here:


www.insightsonindia.com
www.insightsias.com
TEST – 16 Solutions

a) She is the final interpreter of the entire constitution of India within


the house relating to its proceedings.

b) She is the final interpreter only within the house. Outside the house, it
is the courts. So, 1 is wrong.

Statement 3: She decides whether a bill is a money bill or not and his
decision on this question is final.

When a money bill is transmitted to the Rajya Sabha for


recommendation and presented to the President for assent, the Speaker
endorses on the bill his certificate that it is a money bill.

Statement 4: She herself is the chairman of the Business Advisory


Committee, the Rules Committee and the General Purpose Committee.

Q Source: Chapter 22: Indian Polity: M Laxmikanth

31. The institutions of Speaker and Deputy Speaker originated in India


in 1921 under
a) A Charter issued by the British crown for improving
legislative business in British India
b) Provisions of the Government of India Act of 1919
(Montague–Chelmsford Reforms)
c) An order issued by the then Governor-General of India to
manage the Central Legislative Assembly better
d) Amendments made to the Charter Act of 1891 that created
the office of President and Vice-President of Legislative
assemblies

Solution: b)

Learning: At that time, the Speaker and the Deputy Speaker were
called the President and Deputy President respectively and the same
nomenclature continued till 1947.

Before 1921, the Governor- General of India used to preside over the
meetings of the Central Legislative Council.
www.insightsonindia.com
www.insightsias.com
TEST – 16 Solutions

In 1921, the Frederick Whyte and Sachidanand Sinha were appointed by


the Governor-General of India as the first Speaker and the first Deputy
Speaker (respectively) of the central legislative assembly.

In 1925, Vithalbhai J. Patel became the first Indian and the first elected
Speaker of the central legislative assembly.

The Government of India Act o 1935 changed the nomenclatures of


President and Deputy President of the Central Legislative Assembly to
the Speaker and Deputy Speaker respectively.

Q Source: Chapter 22: Indian Polity: M Laxmikanth

32. In reverse repos


a) The banks and financial institutions purchase Government
securities from the RBI
b) Banks and the financial institutions borrow money from the
RBI for the short-term
c) RBI buys commercial papers from the public
d) None of the above

Solution: a)

Justification: Repos allow the banks and the financial institutions to


borrow money from the RBI for the short-term (by selling Government
Securities to the RBI).

In reverse repos, the banks and financial institutions purchase


Government securities from the RBI (basically here the RBI is borrowing
from the banks and the financial institutions).

All the Government securities are dated and the interest for the repo or
reverse repo transactions is announced by the RBI from time to time.

The provision of repos and the reverse repos have been able to serve the
liquidity evenness in the economy as the banks are able to get the
www.insightsonindia.com
www.insightsias.com
TEST – 16 Solutions

required amount of funds out of it, and they can park surplus idle funds
through it.

Q Source: Chapter 11: Ramesh Singh: Indian Economy

33. Each House of Parliament has separate secretarial staff of its


own. The secretariat of each House is headed by a secretary-
general. He is a permanent officer and is appointed by the
a) Leader of the House
b) Presiding Officer of the House
c) President of India
d) The Department of Personnel & Training

Solution: b)
Learning: The Secretariat of Rajya Sabha was set up pursuant to the
provisions contained in Article 98 of the Constitution.

Parliament may by law regulate the recruitment and the conditions of


service of persons appointed to the secretarial staff of either House of
Parliament.

The President may, after consultation with the Speaker of the House of
the People or the Chairman of the Council of States, as the case may be,
make rules regulating the recruitment and the conditions of service of
persons appointed to the secretarial staff of the House.

Q Source: Chapter 22: Indian Polity: M Laxmikanth

34. Which of the following correctly point out the difference(s)


between Adjournment and Prorogation of a house?
1. Unlike Prorogation, Adjournment only terminates a sitting and
not a session of the House.
2. Unlike Adjournment, Prorogation leads to lapse of all the bills
or any other business pending before the House.
www.insightsonindia.com
www.insightsias.com
TEST – 16 Solutions

Select the correct answer using the codes below.

a) 1 only
b) 2 only
c) Both 1 and 2
d) None

Solution: a)
Justification: Statement 1: Prorogation not only terminates a sitting
but also a session of the House.

Statement 2: Prorogation does not affect the bills or any other business
pending before the House. This is the same as adjournment.

However, all pending notices (other than those for introducing bills)
lapse on prorogation and fresh notices have to be given for the next
session.

Q Source: Chapter 22: Indian Polity: M Laxmikanth

35. When the Lok Sabha is dissolved, all business including bills,
motions, resolutions, notices, petitions and so on pending before it
or its committees lapse. However, certain bills do not lapse on the
dissolution of the Lok Sabha. They are?
1. A bill passed by the Lok Sabha but pending in the Rajya Sabha
2. A bill pending in the Rajya Sabha but not passed by the Lok
Sabha
3. A bill passed by both Houses but pending assent of the
president

Select the correct answer using the codes below.

a) 1 and 2 only
b) 2 and 3 only
c) 1 and 3 only
d) 1, 2 and 3

Solution: b)
www.insightsonindia.com
www.insightsias.com
TEST – 16 Solutions

Learning: Some pending bills and all pending assurances that are to be
examined by the Committee on Government Assurances do not lapse on
the dissolution of the Lok Sabha. The position with respect to lapsing of
bills is as follows:

• A bill pending in the Lok Sabha lapses (whether originating in the


Lok Sabha or transmitted to it by the Rajya Sabha).
• A bill passed by the Lok Sabha but pending in the Rajya Sabha
lapses.
• A bill not passed by the two Houses due to disagreement and if the
president has notified the holding of a joint sitting before the
dissolution of Lok Sabha, does not lapse.
• A bill pending in the Rajya Sabha but not passed by the Lok Sabha
does not lapse.
• A bill passed by both Houses but pending assent of the president
does not lapse.
• A bill passed by both Houses but returned by the president for
reconsideration of Houses does not lapse.

Q Source: Chapter 22: Indian Polity: M Laxmikanth

36. A starred question in Parliament, distinguished by an


asterisk
a) Requires an oral answer and hence supplementary questions
can follow
b) Requires a written answer and hence supplementary
questions can follow
c) Any question to which Ministers alone are answerable
d) One that is asked by giving a notice of less than ten days and
answered orally

Solution: a)
Justification: The first hour of every parliamentary sitting is slotted for
the Question hour.
www.insightsonindia.com
www.insightsias.com
TEST – 16 Solutions

During this time, the members ask questions and the ministers usually
give answers. The questions are of three kinds, namely, starred,
unstarred and short notice.

Option (b) is unstarred question.

Option (d) is Short notice questions.

Q Source: Chapter 22: Indian Polity: M Laxmikanth

37. Which of the following statements about the Zero Hour in


Parliament is INCORRECT?
a) The zero hour is not mentioned in the Rules of Procedure of
the house.
b) Matters can be raised by members without prior notice.
c) The zero hour starts before the question hour after which the
agenda for the day is taken up.
d) It is an Indian innovation in the field of parliamentary
procedures and has been in existence since 1962.

Solution: c)
Justification: Unlike the question hour, the zero hour is not mentioned
in the Rules of Procedure. Thus it is an informal device available to the
members of the Parliament to raise matters without any prior notice.

The zero hour starts immediately after the question hour and lasts until
the agenda for the day (ie, regular business of the House) is taken up.

In other words, the time gap between the question hour and the agenda
is known as zero hour.

Q Source: Chapter 22: Indian Polity: M Laxmikanth


www.insightsonindia.com
www.insightsias.com
TEST – 16 Solutions

38. Which of the following correctly point out the difference(s)


between Censure Motion and No Confidence Motion?
1. No-confidence motion should state the reasons for its adoption
in the Lok Sabha, Censure motion need not.
2. No-confidence motion can only be moved against the entire
council of ministers, Censure motion can be moved only against
individual ministers.

Which of the above is/are correct?

a) 1 only
b) 2 only
c) Both 1 and 2
d) None

Solution: d)
Justification: Statement 1: No-confidence motion need not state the
reasons for its adoption in the Lok Sabha, whereas Censure motion
should.

Statement 2: Censure motion can be moved against an individual


minister or a group of ministers or the entire council of ministers. No-
confidence motion can only be moved against the entire council of
ministers. So, 2 is wrong.

Learning: Censure motion is moved for censuring the council of


ministers for specific policies and actions. If it is passed in the Lok
Sabha, the council of ministers need not resign from the office.

No-confidence motion is moved for ascertaining the confidence of Lok


Sabha in the council of ministers. If it is passed in the Lok Sabha, the
council of ministers must resign from office. The motion needs the
support of 50 members to be admitted.

Q Source: Chapter 22: Indian Polity: M Laxmikanth


www.insightsonindia.com
www.insightsias.com
TEST – 16 Solutions

39. The Financial Stability Board (FSB) that promotes


international financial stability through enhanced information
exchange and international cooperation in financial market
supervision was established by
a) The G-8 finance ministers and central bank governors
b) G-20 major economies
c) United Nations Economic and Social Council (UNECOSOC)
d) A resolution of all members of the UN General Assembly

Solution: b)
Learning: The Financial Stability Board (FSB) is an international body
that monitors and makes recommendations about the global financial
system.

It was established after the 2009 G-20 London summit in April 2009 as
a successor to the Financial Stability Forum (FSF).

The Board includes all G-20 major economies, FSF members, and the
European Commission. It is based in Basel, Switzerland

Q Source: Chapter 10: Ramesh Singh: Indian Economy

40. Which of the following statements about Public Bills is


INCORRECT?
1. It is introduced in the Parliament by a minister.
2. It can be introduced without any prior notice in the house.

Which of the above is/are correct?

a) 1 only
b) 2 only
c) Both 1 and 2
d) None

Solution: a)
Justification: Statement 1: Public bills are introduced by Ministers,
whereas private bills are introduced by any Member of Parliament other
than a minister.
www.insightsonindia.com
www.insightsias.com
TEST – 16 Solutions

Statement 2: Introduction of public bill in the House requires seven


days’ notice. Introduction of private bill in the House requires one
month’s notice.

Learning: Public bill reflects of the policies of the government (ruling


party). Its rejection by the House amounts to the expression of want of
parliamentary confidence in the government and may lead to its
resignation.

Q Source: Chapter 22: Indian Polity: M Laxmikanth

41.Article 110 of the Constitution deals with the definition of money


bills. It states that a bill is deemed to be a money bill if it contains
‘only’ provisions dealing with all or any of which of the following
matters?
1. Abolition of a tax
2. Regulation of the borrowing of money by the Union government
3. Appropriation of money out of the Consolidated Fund of India

Select the correct answer using the codes below.

a) 1 and 2 only
b) 2 and 3 only
c) 1 and 3 only
d) 1, 2 and 3

Solution: d)
Learning: It includes the following matters:

• The imposition, abolition, remission, alteration or regulation of


any tax;
• The regulation of the borrowing of money by the Union
government;
• The custody of the Consolidated Fund of India or the contingency
fund of India, the payment of moneys into or the withdrawal of
money from any such fund;
• The appropriation of money out of the Consolidated Fund of India;
www.insightsonindia.com
www.insightsias.com
TEST – 16 Solutions

• Declaration of any expenditure charged on the Consolidated Fund


of India or increasing the amount of any such expenditure;
• The receipt of money on account of the Consolidated Fund of India
or the public account of India or the custody or issue of such
money, or the audit of the accounts of the Union or of a state; or
• Any matter incidental to any of the matters specified above.

However, a bill is not to be deemed to be a money bill by reason only that


it provides for:

• The imposition of fines or other pecuniary penalties, or


• The demand or payment of fees for licenses or fees for services
rendered; or
• The imposition, abolition, remission, alteration or regulation of
any tax by any local authority or body for local purposes.

If any question arises whether a bill is a money bill or not, the decision of
the Speaker of the Lok Sabha is final.

Q Source: Chapter 22: Indian Polity: M Laxmikanth

42. Advanced countries, in particular EU countries and Japan,


have been witnessing a decline in the energy intensity of GDP due
to which of the following factors possibly?
1. Technological improvement
2. Shift in the structure of their economies towards services

Which of the above is/are correct?

a) 1 only
b) 2 only
c) Both 1 and 2
d) None

Solution: c)
www.insightsonindia.com
www.insightsias.com
TEST – 16 Solutions

Justification: Statement 1: Technological improvements reduce energy


consumption per unit of product generated, and there has been a
phenomenal expansion of energy saving technology, for e.g. LED bulbs
replacing CFL bulbs.

Statement 2: Services sector generally consumes less energy per capita


as compared to the industrial sector. So, the shift is more energy saving.
Hence 2 is also correct.

Q Source: Chapter 10: Ramesh Singh: Indian Economy

43. A joint sitting to resolve the Parliamentary deadlock applies


to which of the following types of bills apart from ordinary bills?
1. Appropriation Bill as a part of Annual Budget
2. Constitutional Amendment Bills
3. Financial Bills

Select the correct answer using the codes below.

a) 1 and 2 only
b) 2 and 3 only
c) 3 only
d) 1 only

Solution: c)
Justification: Joint sitting cannot also be called in case of a money bill.

Statement 1: It is a money bill, and hence joint sitting cannot be


summoned.

Statement 2: Each house has the same power with respect to a


constitutional amendment bill, and must pass it separately.

Statement 3: In case of a disagreement between the two Houses over


such a bill, the President can summon a joint sitting of the two Houses to
resolve the deadlock.
www.insightsonindia.com
www.insightsias.com
TEST – 16 Solutions

Q Source: Chapter 22: Indian Polity: M Laxmikanth

44. The expenditure ‘charged’ upon the Consolidated Fund of


India
1. Cannot be discussed by Parliament
2. Is non-votable by the Rajya Sabha, but votable by Lok Sabha
3. Does not require Presidential assent

Select the correct answer using the codes below.

a) 1 and 2 only
b) 1 only
c) 2 and 3 only
d) None of the above

Solution: d)
Justification: Statements 1and 2: The budget consists of two types of
expenditure—the expenditure ‘charged’ upon the Consolidated Fund of
India and the expenditure ‘made’ from the Consolidated Fund of India.

The charged expenditure is non-votable by the Parliament, that is, it can


only be discussed by the Parliament, while the other type has to be voted
by the Parliament.

Statement 3: The final bill containing all such expenditures must receive
Presidential assent to become valid.

Q Source: Chapter 22: Indian Polity: M Laxmikanth

45. The budget goes through six stages in the Parliament.


Arrange the following in the correct order of proceeding in
Parliament.
1. Presentation of budget.
2. Scrutiny by departmental committees.
3. General discussion.
4. Passing of finance bill.
www.insightsonindia.com
www.insightsias.com
TEST – 16 Solutions

5. Passing of appropriation bill.


6. Voting on demands for grants.

Select the correct answer using the codes below.

a) 132564
b) 312654
c) 312456
d) 132654

Solution: d)
Learning: Correct order is:

• Presentation of budget.
• General discussion.
• Scrutiny by departmental committees.
• Voting on demands for grants.
• Passing of appropriation bill.
• Passing of finance bill.

The general discussion on budget begins a few days after its


presentation. It takes place in both the Houses of Parliament and lasts
usually for three to four days.

After the general discussion on the budget is over, the Houses are
adjourned for about three to four weeks and 24 departmental standing
committees examine the budget.

After voting demand for grants, an appropriation bill is introduced to


provide for the appropriation, out of the Consolidated Fund of India, all
money required to meet:

• The grants voted by the Lok Sabha.


• The expenditure charged on the Consolidated Fund of India.

Q Source: Chapter 22: Indian Polity: M Laxmikanth

46. ‘Vote on account’ provision is related to


www.insightsonindia.com
www.insightsias.com
TEST – 16 Solutions

a) Any grant made in advance by the Lok Sabha to the executive


before the passing of appropriation bill
b) Excess grant awarded by the Parliament to the executive that
had been appropriated in the annual budget
c) Grant made before the ‘March Rush’ by the Lok Sabha to the
executive
d) Grant sanctioned by Parliament to executive for meeting an
unexpected demand upon the resources of India

Solution: a)
Learning: The Appropriation Bill becomes the Appropriation Act after
it is assented to by the President.

This act authorises (or legalises) the payments from the Consolidated
Fund of India. This means that the government cannot withdraw money
from the Consolidated Fund of India till the enactment of the
appropriation bill. This takes time and usually goes on till the end of
April.

• But the government needs money to carry on its normal activities


after 31 March (the end of the financial year).
• To overcome this functional difficulty, the Constitution has
authorised the Lok Sabha to make any grant in advance in respect
to the estimated expenditure for a part of the financial year,
pending the completion of the voting of the demands for grants
and the enactment of the appropriation bill.
• This provision is known as the ‘vote on account’. It is passed (or
granted) after the general discussion on budget is over.
• It is generally granted for two months for an amount equivalent to
one-sixth of the total estimation.

Q Source: Chapter 22: Indian Polity: M Laxmikanth

47. Which of the following statements with reference to the


Public Account of India is CORRECT?
a) It is operated by executive action.
www.insightsonindia.com
www.insightsias.com
TEST – 16 Solutions

b) All taxes collected by the Government of India are submitted


to this account.
c) It is used to fund contingencies and disaster management.
d) It is an extra-constitutional fund.

Solution: a)
Justification & Learning: All other public money (other than those
which are credited to the Consolidated Fund of India) received by or on
behalf of the Government of India shall be credited to the Public Account
of India.

This includes provident fund deposits, judicial deposits, savings bank


deposits, departmental deposits, remittances and so on. This account is
operated by executive action, that is, the payments from this account can
be made without parliamentary appropriation.

Such payments are mostly in the nature of banking transactions.

Q Source: Chapter 22: Indian Polity: M Laxmikanth

48. Which of the following is the authority to create or abolish


the state legislative councils?
a) President of India
b) Union Council of Ministers
c) Parliament
d) Governor of the Concerned State

Solution: c)
Learning: The question in hand shows the unitary tendencies of the
Indian federation cum Union.

The Parliament can create or abolish the state legislative councils on the
recommendation of the concerned state legislative assemblies.

It is notable that these councils are created by the Parliament too.


www.insightsonindia.com
www.insightsias.com
TEST – 16 Solutions

It can also increase or decrease the area, alter the boundaries and change
the names of states of the Indian Union.

Q Source: Chapter 22: Indian Polity: M Laxmikanth

49. In 2014, Supreme court of India setup a Committee under


NR Madhava Menon to frame guidelines for
a) Allowing new mining operations in left wing extremism
affected areas
b) Government advertisements in print and media
c) Regulation of economic activities in Ecologically Sensitive
Zones (ESZs)
d) Improving the socio-economic status of the Transgender

Solution: b)
Learning: To keep politics away from such ads, the committee
emphasized that only the pictures and names of the President, the Prime
Minister, Governor and Chief Ministers should be published.

• The committee also endorsed the suggestions of the Election


Commission that there must be severe restrictions on such
advertisements six months prior to election.
• Apart from this, the committee also recommended that a deadline
should be fixed to prohibit their publication and the poll panel
should be authorised for the purpose.
• The committee also recommended that there should only be a
single advertisement, preferably by Information and Broadcasting
Ministry, in respect of commemorative advertisements, which are
given on birth and death anniversary of an important personality.

Q Source: SC allows pictures of CMs, Governors, Ministers in


Government Advertisements
www.insightsonindia.com
www.insightsias.com
TEST – 16 Solutions

50. Consider the following statements about a certain


Parliamentary Committee.
1. The Rajya Sabha has no representation in this committee.
2. The committee examine the estimates included in the budget
and suggest ‘economies’ in public expenditure.
3. The committee works throughout the financial year and reports
to the House as its examination proceeds.

The above refer to?

a) Committee on Public Undertakings


b) Departmental Standing Committees
c) Committee on Government Finances
d) Estimates Committee

Solution: d)
Learning: All the thirty members are from Lok Sabha only.

These members are elected by the Lok Sabha every year from amongst
its members.

• A minister cannot be elected as a member of the committee. The


chairman of the committee is appointed by the Speaker from
amongst its members and he is invariably from the ruling party.
• The function of the committee is to examine the estimates included
in the budget and suggest ‘economies’ in public expenditure.
Hence, it has been described as a ‘continuous economy committee’.
• It is not incumbent on the committee to examine the entire
estimates of any one year. The demands for grants are finally voted
despite the fact that the committee has made no report.

Q Source: Chapter 22: Indian Polity: M Laxmikanth

51. Consider the following statements.


www.insightsonindia.com
www.insightsias.com
TEST – 16 Solutions

Assertion (A): The single system of courts that enforces both Central
laws as well as the state laws has been adopted in India from the
Government of India Act of 1935.

Reason (R): The Government of India Act of 1935 established the


Federal Court of India with original, appellate and advisory jurisdiction.

In the context of the above, which of these is correct?

a) A is correct, and R can be an appropriate explanation of A.


b) A is correct, but R cannot be an appropriate explanation of A.
c) A is correct, but R is incorrect.
d) A is incorrect, but R is correct.

Solution: a)
Justification: The Federal Court of India was a judicial body,
established in India in 1937 under the provisions of the Government of
India Act 1935, with original, appellate and advisory jurisdiction.

It functioned until 1950, when the Supreme Court of India was


established.

The Federal Court had exclusive original jurisdiction in any dispute


between the Central Government and the Provinces.

Initially, it was empowered to hear appeals from the High Courts of the
provinces in the cases which involved the interpretation of any Section of
the Government of India Act, 1935.

Q Source: Chapter 25: Indian Polity: M Laxmikanth

52. A dispute between two or more states will fall under Supreme
Court’s
a) Exclusive Original Jurisdiction
b) Appellate Jurisdiction
c) Writ Jurisdiction
www.insightsonindia.com
www.insightsias.com
TEST – 16 Solutions

d) Advisory Jurisdiction

Solution: a)
Learning: As a federal court, the Supreme Court decides the disputes
between different units of the Indian Federation. More elaborately, any
dispute between:

• The Centre and one or more states; or


• The Centre and any state or states on one side and one or more
states on the other; or
• Between two or more states.

In the above federal disputes, the Supreme Court has exclusive original
jurisdiction.

Exclusive means, no other court can decide such disputes and original
means, the power to hear such disputes in the first instance, not by way
of appeal.

Q Source: Chapter 25: Indian Polity: M Laxmikanth

53. The Supreme Court enjoys appellate jurisdiction in


1. Civil matters
2. Criminal matters
3. Constitutional matters

Select the correct answer using the codes below.

a) 1 and 2 only
b) 2 and 3 only
c) 1 and 3 only
d) 1, 2 and 3

Solution: d)
Learning: The Supreme Court is primarily a court of appeal and hears
appeals against the judgements of the lower courts.
www.insightsonindia.com
www.insightsias.com
TEST – 16 Solutions

It enjoys a wide appellate jurisdiction which can be classified under four


heads:

• Appeals in constitutional matters.


• Appeals in civil matters.
• Appeals in criminal matters.
• Appeals by special leave.

In the constitutional cases, an appeal can be made to the Supreme Court


against the judgement of a high court if the high court certifies that the
case involves a substantial question of law that requires the
interpretation of the Constitution.

Q Source: Chapter 25: Indian Polity: M Laxmikanth

54. The constitutional validity of a legislative enactment or an


executive order can be challenged in the Supreme Court on which
of the following grounds?
1. If it infringes the Fundamental Rights
2. If it is repugnant to the constitutional provisions
3. If it is outside the competence of the authority which has framed
it

Select the correct answer using the codes below.

a) 1 and 2 only
b) 2 and 3 only
c) 1 and 3 only
d) 1, 2 and 3

Solution: d)
Learning: Judicial review is the power of the Supreme Court to
examine the constitutionality of legislative enactments and executive
orders of both the Central and state governments.

On examination, if they are found to be violative of the Constitution or


fundamental rights (ultra-vires), they can be declared as illegal,
www.insightsonindia.com
www.insightsias.com
TEST – 16 Solutions

unconstitutional and invalid (null and void) by the Supreme Court.


Consequently, they cannot be enforced by the Government.

Judicial review is needed for the following reasons:

• To uphold the principle of the supremacy of the Constitution.


• To maintain federal equilibrium (balance between Centre and
states).
• To protect the fundamental rights of the citizens

Q Source: Chapter 25: Indian Polity: M Laxmikanth

55. How is the Governor of a state appointed?


a) Indirectly elected by the State Legislative Assembly
b) Nominated by the President
c) Elected by a collegium consisting of the heads of all local
bodies of the concerned State
d) Nominated by the State Government subject to the approval
of the Union Government

Solution: b)
Justification & Learning: The governor is neither directly elected by
the people nor indirectly elected by a specially constituted electoral
college as is the case with the president.

He is appointed by the president by warrant under his hand and seal. In


a way, he is a nominee of the Central government. But, as held by the
Supreme Court in 1979, the office of governor of a state is not an
employment under the Central government.

It is an independent constitutional office and is not under the control of


or subordinate to the Central government.

Q Source: Chapter 26: Indian Polity: M Laxmikanth


www.insightsonindia.com
www.insightsias.com
TEST – 16 Solutions

56. Consider the following about the terms of the Office of the
Governor.
1. The Governor can resign at any time by addressing a resignation
letter to the Chief Minister of the State.
2. The Constitution does not lay down the grounds upon which a
governor may be removed by the President.

Which of the above is/are correct?

a) 1 only
b) 2 only
c) Both 1 and 2
d) None

Solution: b)
Justification: Statement 1: The resignation letter is submitted to the
President, and not the Chief Minister. So, 1 is wrong.

Statement 2: A governor holds office for a term of five years from the
date on which he enters upon his office. However, this term of five years
is subject to the pleasure of the President.

The Supreme Court held that the pleasure of the President is not
justifiable. The governor has no security of tenure and no fixed term of
office. He may be removed by the President at any time without
mentioning any grounds for his removal.

Q Source: Chapter 26: Indian Polity: M Laxmikanth

57. Before entering upon his office, the President has to make
and subscribe to an oath or affirmation. Which of the following
is/are part of the oath or affirmation made by the President?
1. To preserve, protect and defend the Constitution and the law
2. To uphold the sovereignty and integrity of India
www.insightsonindia.com
www.insightsias.com
TEST – 16 Solutions

3. Perform the duties of the office without fear or favour

Select the correct answer using the codes below.

a) 1 and 2 only
b) 2 and 3 only
c) 1 only
d) 1, 2 and 3

Solution: c)
Justification: In his oath, the President swears:

• to faithfully execute the office;

• to preserve, protect and defend the Constitution and the law; and

• to devote himself to the service and well-being of the people of


India.

The oath of office to the President is administered by the Chief Justice of


India.

Oath in Statement 2 is taken by the judges of Supreme Court and High


Courts, and

Oath in Statement 2 is taken by the information commissioners, judges


of Supreme Court and High Courts etc.

Q Source: Chapter 17: Indian Polity: M Laxmikanth

58. How is the Lok Sabha speaker chosen?


a) Nominated by a collegium of Prime Minister, Leader of
Opposition and Minister for Parliamentary affairs
b) Nominated by the party/coalition in majority in Lok Sabha
c) Elected by the house from a body of eminent citizens
specially selected for this purpose
d) Elected by the house from amongst its members
www.insightsonindia.com
www.insightsias.com
TEST – 16 Solutions

Solution: d)
Learning: The Speaker is elected by the Lok Sabha from amongst its
members (as soon as may be, after its first sitting).

Whenever the office of the Speaker falls vacant, the Lok Sabha elects
another member to fill the vacancy. The date of election of the Speaker is
fixed by the President.

Usually, the Speaker remains in office during the life of the Lok Sabha.

Q Source: Chapter 22: Indian Polity: M Laxmikanth

59. Which of the following is/are the differences between the


pardoning Powers of President and Governor?
1. Governor cannot pardon a death sentence, President can.
2. Governor can pardon a sentence of court martial, President
cannot.

Which of the above is/are correct?

a) 1 only
b) 2 only
c) Both 1 and 2
d) None

Solution: a)
Justification: Statement 1: Even if a state law prescribes for death
sentence, the power to grant pardon lies with the President and not the
governor. But, the governor can suspend, remit or commute a death
sentence.

Statement 2: The President can grant pardon, reprieve, respite,


suspension, remission or commutation in respect to punishment or
sentence by a court-martial (military court). Governor cannot.

Q Source: Chapter 26: Indian Polity: M Laxmikanth


www.insightsonindia.com
www.insightsias.com
TEST – 16 Solutions

60. The Governor has constitutional discretion in which of the


following cases?
1. Recommendation for the imposition of the President’s Rule in
the state
2. While exercising his functions as the administrator of an
adjoining union territory
3. Seeking information from the chief minister with regard to the
administrative and legislative matters of the state
4. Reservation of a bill for the consideration of the President.

Select the correct answer using the codes below.

a) 1 and 2 only
b) 3 and 4 only
c) 1, 3 and 4 only
d) 1, 2, 3 and 4

Solution: d)
Learning: One other constitutional discretion is determining the
amount payable by the Government of Assam, Meghalaya, Tripura and
Mizoram to an autonomous Tribal District Council as royalty accruing
from licenses for mineral exploration.

In addition to the above constitutional discretion (i.e., the express


discretion mentioned in the Constitution), the governor, like the
president, also has situational discretion (i.e., the hidden discretion
derived from the exigencies of a prevailing political situation) in the
following cases:

• Appointment of chief minister when no party has a clear-cut


majority in the state legislative assembly or when the chief
minister in office dies suddenly and there is no obvious successor.
• Dismissal of the council of ministers when it cannot prove the
confidence of the state legislative assembly.
• Dissolution of the state legislative assembly if the council of
ministers has lost its majority.
www.insightsonindia.com
www.insightsias.com
TEST – 16 Solutions

It is to be noted that the President has no or little constitutional


discretion.

Q Source: Chapter 26: Indian Polity: M Laxmikanth

61.The Union Commerce and Industry Ministry has notified foreign


direct investment (FDI) upto 49% in insurance and pension sector
will be under automatic route. What implication it will have for the
sectors?
1. The Indian corporate laws would not apply to companies
investing within the 49% FDI limit.
2. Foreign Companies investing in these sectors within the cap
would not require prior approval from government.

Which of the above is/are correct?

a) 1 only
b) 2 only
c) Both 1 and 2
d) None

Solution: b)
Justification: An Indian company may receive Foreign Direct
Investment under the two routes as given under:

i. Automatic Route

FDI is allowed under the automatic route without prior approval either
of the Government or the Reserve Bank of India in all activities/sectors
as specified in the consolidated FDI Policy, issued by the Government of
India from time to time.

ii. Government Route

FDI in activities not covered under the automatic route requires prior
approval of the Government which are considered by the Foreign
Investment Promotion Board (FIPB), Department of Economic Affairs,
Ministry of Finance.
www.insightsonindia.com
www.insightsias.com
TEST – 16 Solutions

So, clearly statement 2 is correct. Statement 1 is incorrect as corporate


laws will apply irrespective of the FDI coming in through automatic or
government route.

Q Source: http://www.dnaindia.com/money/report-dipp-notifies-49-
fdi-under-automatic-route-in-insurance-pension-sector-2193287

62. Consider the following about the ordinance-making Power of


the President.
1. The ordinances have the same force and effect as an act of
Parliament.
2. An ordinance can be issued even if only one House of
Parliament is in session.
3. It can be issued to amend the Constitution.
4. It is subject to judicial review.

Select the correct answer using the codes below.

a) 1, 3 and 4 only
b) 1 only
c) 1, 2 and 4 only
d) 2 and 3 only

Solution: c)
Justification: Statement 1 and 3: An ordinance like any other
legislation, can be retrospective, that is, it may come into force from a
back date. It may modify or repeal any act of Parliament or another
ordinance. It can alter or amend a tax law also. However, it cannot be
issued to amend the Constitution.

Statement 2: An ordinance can also be issued when only one House is in


session because a law can be passed by both the Houses and not by one
House alone. An ordinance made when both the Houses are in session is
void.
www.insightsonindia.com
www.insightsias.com
TEST – 16 Solutions

Statement 3: He can make an ordinance only when he is satisfied that


the circumstances exist that render it necessary for him to take
immediate action. In Cooper case, (1970), the Supreme Court held that
the President’s satisfaction can be questioned in a court on the ground of
malafide.

Q Source: Chapter 17: Indian Polity: M Laxmikanth

63. The president nominates 12 members to the Rajya Sabha


from
a) The persons recommended by the National Integration
Council
b) People who have special knowledge or practical experience in
art, literature, science and social service.
c) People who have contributed to immensely to Indian politics.
d) Eminent political scientists who have never contested an
election

Solution: b)
Learning: The rationale behind this principle of nomination is to
provide eminent persons a place in the Rajya Sabha without going
through the process of election.

Some of these members currently are Sachin Tendulkar, Anu Aga, K.


Prasaran, K.T.S. Tulsi.

It should be noted here that the American Senate has no nominated


members.

Q Source: Chapter 22: Indian Polity: M Laxmikanth


www.insightsonindia.com
www.insightsias.com
TEST – 16 Solutions

64. Nepal has recently signed an agreement with China to build a


strategic railway link between the two countries through Tibet.
What implication does it have for India?
1. It may reduce Nepal’s dependency on Indian sea ports.
2. It may reduce India’s dominance of Nepal’s overall trade with
Asia.

Which of the above is/are correct?

a) 1 only
b) 2 only
c) Both 1 and 2
d) None

Solution: c)
Learning: Presently, India has virtual monopoly in trade with the
Nepal which accounts for one third trade of the Himalayan landlocked
country.

The agreements signed also include a feasibility study on the


establishment of a Free Trade Agreement (FTA) between both countries.

China will also provide assistance to build a border bridge over the
Simikot-Hilsa road section that will connect Humla district of Nepal with
Tibet.

The article in the Q Source explains the intricacies of the important issue
well.

Q Source: http://www.thehindu.com/news/international/nepal-inks-
transit-treaty-with-china-to-have-first-rail-link/article8381195.ece

65. Part V of the Indian constitution deals with which of the


following matters?
a) Organisation, privileges and powers of the Parliament
b) Special provisions relating to certain classes
www.insightsonindia.com
www.insightsias.com
TEST – 16 Solutions

c) Relations between the Union and the States


d) Provisions for acquisition and termination of citizenship

Solution: a)
Learning: Articles 79 to 122 in Part V of the Constitution deal with the
organisation, composition, duration, officers, procedures, privileges,
powers and so on of the Parliament.

Under the Constitution, the Parliament of India consists of three parts


viz, the President, the Council of States and the House of the People.

Though the President of India is not a member of either House of


Parliament and does not sit in the Parliament to attend its meetings, he
is an integral part of the Parliament.

Q Source: Chapter 22: Indian Polity: M Laxmikanth

66. Consider the following statements about the role of the


Prime Minister.
1. He allocates and reshuffles various portfolios among the
ministers.
2. He can bring about the collapse of the council of ministers by
resigning from office.

Which of the above is/are correct?

a) 1 only
b) 2 only
c) Both 1 and 2
d) None

Solution: c)
www.insightsonindia.com
www.insightsias.com
TEST – 16 Solutions

Learning: Since the Prime Minister stands at the head of the council of
ministers, the other ministers cannot function when the Prime Minister
resigns or dies.

In other words, the resignation or death of an incumbent Prime Minister


automatically dissolves the council of ministers and thereby generates a
vacuum.

The resignation or death of any other minister, on the other hand,


merely creates a vacancy which the Prime Minister may or may not like
to fill.

Q Source: Chapter 19: Indian Polity: M Laxmikanth

67. The Union Ministry of Health and Family Welfare has


recently amended the Schedule V of the Drugs and Cosmetics
Rules, 1945. As per the new rules issued by the Ministry
a) Clinical trials in India will be banned.
b) Prior government permission will be needed for clinical
trials.
c) Clinical trials will be banned for academic purposes.
d) None of the above

Solution: d)
Justification: The new rules state that:

• There is no need for repeat tests on animals for new drug


registrations if complete data from similar toxicity experiments
already exists for drugs approved abroad.
• However, no permission will be required to conduct clinical trial
intended for academic purposes in respect of approved drug
formulation
www.insightsonindia.com
www.insightsias.com
TEST – 16 Solutions

The amendments come after repeated appeals by People for the Ethical
Treatment of Animals (PETA) India and Union Women & Child
Development Minister Maneka Gandhi who is a well-known animal
rights activist.

Q Source: http://www.newindianexpress.com/nation/Centre-Bans-
Repeat-Drug-Tests-on-Animals/2016/03/19/article3334598.ece

68. Which of the following statements about the Public Accounts


Committee is INCORRECT?
a) Its members are nominated by the Speaker giving due
representation from all parties.
b) The term of office of the members is one year.
c) A minister cannot be chosen as a member of the committee.
d) The committee examines the annual audit reports of the
comptroller and auditor general of India (CAG).

Solution: a)
Justification: At present, it consists of 22 members (15 from the Lok
Sabha and from the Rajya Sabha).

The members are elected by the Parliament every year from amongst its
members according to the principle of proportional representation by
means of the single transferable vote. Thus, all parties get due
representation in it.

The chairman of the committee is appointed by the Speaker from


amongst its members.

Until 1966–67, the chairman of the committee belonged to the ruling


party. However, since 1967 a convention has developed whereby the
chairman of the committee is selected invariably from the Opposition

Q Source: Chapter 22: Indian Polity: M Laxmikanth


www.insightsonindia.com
www.insightsias.com
TEST – 16 Solutions

69. ‘Drishti’ deployed by the Indian Metrological Department


helps
a) Provide real time visibility information to pilots
b) Provide quick weather forecasts to farmers
c) Detect early cyclones and tsunamis in coastal areas
d) Track the buildup of Monsoon in Northern region

Solution: a)
Learning: Generally due to the poor visibility, the aircraft operations
would often get disturbed that affects passenger traffic during winter and
monsoon seasons especially in Northern India.

Drishti is a transmissometer that takes in account the real time weather


parameters like humidity, wind speed, temperature conditions, etc.,
along the runway in order to calculate the visibility distance offered at
that particular runway.

It is cost-effective mandatory system required at all airports as per


International Civil Aviation Organisation (ICAO) and World
Meteorological Organisation (WMO).

Q Source: http://indianexpress.com/article/cities/mumbai/made-in-
india-visibility-device-to-be-installed-at-mumbai-airport-soon/

70. To be eligible for election as Vice-President, a person should


fulfil which of the following qualifications?
1. He should be qualified for election as a member of the Rajya
Sabha.
2. He should not be affiliated to a political party.
3. He should not have ever held the office of the President.

Select the correct answer using the codes below.

a) 1 only
b) 2 and 3 only
c) 1 and 3 only
d) 1 and 2 only
www.insightsonindia.com
www.insightsias.com
TEST – 16 Solutions

Solution: a)
Justification: Statement 1: Article 102 of the Constitution lays down
that a person shall be disqualified for being chosen as, and for being, a
member of either House of Parliament, e.g. unsound mind, undischarged
insolvent etc.

So, the nominee to the office of the Vice-President must fulfil these
conditions apart from others.

Statement 2 and 3: He should not hold any office of profit under the
Union government or any state government or any local authority or any
other public authority. Conditions in 2 and 3 do not prevent him/her to
file nomination for election to the office of the Vice-President.

Even a sitting President or Vice-President of the Union, the governor of


any state and a minister for the Union or any state is not deemed to hold
any office of profit and hence qualified for being a candidate for Vice-
President.

Q Source: Chapter 18: Indian Polity: M Laxmikanth

71. “Yugantar Ashram” is best known as the


a) First war memorial established by the British in respect of
Indian freedom fighters
b) Centre of Indian National Congress’s agitations
c) Headquarters of the Gadar party
d) Hub of mutiny and revolutionary thought in India

Solution: c)
Learning: The Hindustan Association of the Pacific Coast, known as
the Gadar Party was founded in 1913 to free India from British slavery.
The headquarters of the association was established initially at 436 Hill
Street, San Francisco and named as “Yugantar Ashram.”

The GoI decided in 2013 to convert this memorial into a library and
Museum.
www.insightsonindia.com
www.insightsias.com
TEST – 16 Solutions

http://www.sikhfoundation.org/people-events/gadar-memorial-in-san-
francisco-to-be-museum-and-library/

Q Source: Chapter 12: Bipin Chandra: India’s Struggle for


Independence

72. Kutiyattam is a form of


a) Folk art originating from the early Tamil country
b) Sanskrit theatre traditionally performed in the state of Kerala
c) Dance performed by female members of the Luriyum
community of Andhra Pradesh
d) Poetic performance accompanied by a Mridangam in South
India

Solution: b)
Learning: It is officially recognised by UNESCO as a Masterpiece of the
Oral and Intangible Heritage of Humanity.

Traditionally, the main musical instruments used in Koodiyattam are


mizhavu, kuzhitalam, etakka, kurumkuzhal, and sankhu. Mizhavu, the
most prominent of these, is a percussion instrument that is played by a
person of the Ambalavas Nambiar caste, accompanied by Nangyaramma
playing the kuzhithalam.

Traditionally, Koodiyattam has been performed by Chakyars (a subcaste


of Kerala Hindus) and by Nangyaramma (women of the Ambalavasi
Nambiar caste).

Q Source: Chapter 5: India Yearbook 2016


www.insightsonindia.com
www.insightsias.com
TEST – 16 Solutions

73. The question of disqualification under the Anti-defection


provisions under Tenth Schedule of the constitution is decided by
the
a) Chairman in the case of Rajya Sabha and Speaker in the case
of Lok Sabha
b) Supreme Court in case of Parliament and concerned High
courts in case of State legislative assemblies
c) President of India in all cases based on the recommendations
of the Union cabinet
d) Election Commission of India

Solution: a)
Learning: The Constitution lays down that a person shall be
disqualified from being a member of Parliament if he is so disqualified
on the ground of defection under the provisions of the Tenth Schedule. A
member incurs disqualification under the defection law:

• If he voluntary gives up the membership of the political party on


whose ticket he is elected to the House;

• If he votes or abstains from voting in the House contrary to any


direction given by his political party;

• If any independently elected member joins any political party; and

• If any nominated member joins any political party after the expiry
of six months.

The question of disqualification under the Tenth Schedule is decided by


the Chairman in the case of Rajya Sabha and Speaker in the case of Lok
Sabha (and not by the president of India).

In 1992, the Supreme Court ruled that the decision of the


Chairman/Speaker in this regard is subject to judicial review.

Q Source: Chapter 22: Indian Polity: M Laxmikanth


www.insightsonindia.com
www.insightsias.com
TEST – 16 Solutions

74. Which of the following fellowships are awarded by Sahitya


Akademi?
1. Sahitya Akademi Honorary Fellowhip
2. Anand Fellowship
3. Rabindranath Tagore Fellowship

Select the correct answer using the codes below.

a) 1 and 2 only
b) 2 and 3 only
c) 1 and 3 only
d) 1, 2 and 3

Solution: a)
Learning: The third fellowship (statement 3) is Premchand fellowship.

The Akademi confers its fellowship, its highest honour to literary figures.
Sahitya Adademi also instituted a fellowship named after Premchand
during his 125th Birth Anniversary in 2005 for scholars doing research
on Indian literature or to creative writers from the countries of the
SAARC region other than India.

Every year since its inception in 1954, the Sahitya Akademi awards prizes
to the most outstanding books of literary merit published in any of the
major Indian languages recognized by the Akademi.

Q Source: Chapter 5: India Yearbook 2016

75. Consider the following about the powers of the President of


India.
1. He can appoint an inter-state council to promote Centre–state
and inter-state cooperation.
2. He directly administers the union territories through
administrators appointed by him.
www.insightsonindia.com
www.insightsias.com
TEST – 16 Solutions

3. He can declare an area as scheduled area and has powers with


respect to the administration of scheduled areas and tribal
areas.

Select the correct answer using the codes below.

a) 1 and 2 only
b) 2 and 3 only
c) 1 and 3 only
d) 1, 2 and 3

Solution: d)
Learning: Apart from the above functions, he also has the following
powers:

• He can seek any information relating to the administration of


affairs of the Union, and proposals for legislation from the prime
minister.
• He can require the Prime Minister to submit, for consideration of
the council of ministers, any matter on which a decision has been
taken by a minister but, which has not been considered by the
council.
• He can appoint a commission to investigate into the conditions of
SCs, STs and other backward classes.

Q Source: Chapter 17: Indian Polity: M Laxmikanth

76. Consider the following about the India Tourism


Development Corporation Limited (ITDC).
1. It is owned by the Government of India and functions under the
Ministry of Culture.
2. It can develop tourism infrastructure through the public-private
partnership (PPP) mode in the country.
www.insightsonindia.com
www.insightsias.com
TEST – 16 Solutions

Which of the above is/are correct?

a) 1 only
b) 2 only
c) Both 1 and 2
d) None

Solution: b)
Justification: Statement 1: The India Tourism Development
Corporation Limited (ITDC) is an Hospitality, retail and Education
company owned by Government of India, under Ministry of Tourism.

Established in 1966, it owns over 17 properties under the Ashok Group of


Hotels brand, across India.

Statement 2: Indian Tourism Development Corporation Ltd (ITDC) has


set up a new joint venture (JV) with IL&FS to develop tourism
infrastructure through the public-private partnership (PPP) mode in the
country.

The equal JV will be called Ashok Tourism Infrastructure Development


(Ashok Infra).

Q Source: Chapter 5: India Yearbook 2016

77.Consider the following about the trends of National Income in


India.
1. India’s GDP growth rate has always increased post-2004.
2. India’s GDP has never shrunk since 1991 reforms.

Which of the above is/are correct?

a) 1 only
b) 2 only
c) Both 1 and 2
d) None

Solution: b)
www.insightsonindia.com
www.insightsias.com
TEST – 16 Solutions

Justification: Statement 1: It had slowed down during world recession


and euro zone slowdown post-2008. So, 1 is incorrect.

Statement 2: The growth rate of India’s GDP has always been positive
even if it fluctuated since 1991. So, GDP had never shrunk since 1991.

You can see the trends here.

http://statisticstimes.com/economy/gdp-of-india.php

Q Source: Chapter 6: India Yearbook 2016

78. Consider the following statements.

Assertion (A): India is the largest exporter in commercial services.

Reason (R): Services sector contributes to more than 50% of India’s


GDP.

In the context of the above, which of these is correct?

a) A is correct, and R is an appropriate explanation of A.


b) A is correct, but R is not an appropriate explanation of A.
c) A is correct, but R is incorrect.
d) A is incorrect, but R is correct.

Solution: d)
Learning: India was the eighth largest exporter in commercial services
with 3.2 per cent share of world exports and the tenth largest importer
with 2.6 per cent share of world imports in 2014. So, clearly A is wrong.

Services sector forms around 65% id India’s GDP. SO, R is correct.

Q Source: Chapter 6: India Yearbook 2016


www.insightsonindia.com
www.insightsias.com
TEST – 16 Solutions

79. Which of the following sectors is/are included in the Index of


industrial production (IIP)?
1. Mining
2. Electricity
3. Fertilizers

Select the correct answer using the codes below.

a) 1 and 2 only
b) 2 and 3 only
c) 1 and 3 only
d) 1, 2 and 3

Solution: d)
Learning: The Index of Industrial Production (IIP) is an index for India
which details out the growth of various sectors in an economy such as
mining, electricity and manufacturing. It includes all 8 core industries
including steel, fertilizers etc.

The all India IIP is a composite indicator that measures the short-term
changes in the volume of production of a basket of industrial products
during a given period with respect to that in a chosen base period.

It is compiled and published monthly by the Central Statistical


Organisation (CSO) six weeks after the reference month ends.

Q Source: Chapter 6: India Yearbook 2016

80. The Union executive consists of the


1. President
2. Vice-President
3. Prime Minister
4. Council of Ministers
5. Attorney-General of India
www.insightsonindia.com
www.insightsias.com
TEST – 16 Solutions

6. Central Administrative Tribunal


7. Central Vigilance Commission

Select the correct answer using the codes below.

a) 1, 2, 3, 4 and 5 only
b) 1, 2, 5, 6 and 7 only
c) 3 and 4 only
d) 1, 2, 3, 4, 5, 6 and 7

Solution: a)
Justification: Statement 6: CAT is tribunal wielding judicial powers,
and cannot be said to be a part of the Union executive.

Statement 7: CVC acts as a quasi-judicial body that inquires into cases of


corruption in the Union government.

Articles 52-78 (Part V) of the Constitution deal with the Union executive.

President heads the Union executive.

Q Source: Chapter 17: Indian Polity: M Laxmikanth

81. India has signed a free trade agreement (FTA) with which of
these nations?
1. Sri Lanka
2. China
3. European Union

Select the correct answer using the codes below.

a) 1 and 2 only
b) 1 only
c) 1 and 3 only
d) 3 only

Solution: b)
www.insightsonindia.com
www.insightsias.com
TEST – 16 Solutions

Learning: Such agreements involve cooperation between India and the


partner country to reduce trade barriers – import quotas and tariffs –
and to increase trade of goods and services with each other.

Here is the list


http://commerce.nic.in/trade/international_ta.asp?id=2&trade=i

Q Source: Chapter 7: India Yearbook 2016

82. The main objectives of the Special Economic Zone (SEZ) Act,
2005 are
1. Promotion of exports of goods and services
2. Promotion of investment from foreign sources
3. Development of infrastructure facilities

Select the correct answer using the codes below.

a) 1 and 2 only
b) 2 and 3 only
c) 1 and 3 only
d) 1, 2 and 3

Solution: d)
Learning: Special Economic Zone (SEZ) is a specifically delineated
duty-free enclave and shall be deemed to be foreign territory for the
purposes of trade operations and duties and tariffs.

In order words, SEZ is a geographical region that has economic laws


different from a country's typical economic laws.

Usually the goal is to increase foreign investments. SEZs have been


established in several countries, including China, India, Jordan, Poland,
Kazakhstan, Philippines and Russia. North Korea has also attempted this
to a degree.

Q Source: Chapter 7: India Yearbook 2016


www.insightsonindia.com
www.insightsias.com
TEST – 16 Solutions

83. Consider the following about the impeachment of President


of India.
1. The impeachment charges can be initiated by either House of
Parliament.
2. The Parliament investigates the impeachment charges.
3. No President has so far been impeached.

Select the correct answer using the codes below.

a) 1 and 2 only
b) 2 and 3 only
c) 1 only
d) 1, 2 and 3

Solution: d)
Justification: Statement 1: Impeachment is a quasi-judicial procedure
in the Parliament. These charges should be signed by one-fourth
members of the House (that framed the charges).

Statement 2: After the impeachment resolution is passed by a majority


of two-thirds of the total membership of that House, it is sent to the
other House, which should investigate the charges.

The President has the right to appear and to be represented at such


investigation.

If the other House also sustains the charges and passes the impeachment
resolution by a majority of two-thirds of the total membership, then the
President stands removed from his office from the date on which the bill
is so passed.

Q Source: Chapter 17: Indian Polity: M Laxmikanth


www.insightsonindia.com
www.insightsias.com
TEST – 16 Solutions

84. Project Arrow related with Service Quality Improvement is


an initiative of
a) Department of Posts
b) Ministry of Defence
c) Ministry of Tourism
d) Ministry of Tribal Welfare

Solution: a)
Learning: Project Arrow entails comprehensive improvement of the
‘core operations’ of the Post Offices as well as modernized ambience
under ‘Look & Feel’ silo of the project.

Enhancing the quality of services in ‘core areas’ envisages focus on mail


delivery, remittances, savings bank and office service levels through
continuous monitoring of Key Performance Indicators, identifying
problem areas and take remedial action on a real-time basis, including
making systemic changes to improve service quality.

Q Source: Chapter 8: India Yearbook 2016

85. The system of proportional representation aims at removing


the defects of territorial representation, where all sections of the
people get representation in proportion to their number. What
types of proportional representation are followed in India?
1. List system
2. Single transferable vote system
3. Mixed member system

Select the correct answer using the codes below.

a) 1 and 2 only
b) 2 only
c) 1 and 3 only
www.insightsonindia.com
www.insightsias.com
TEST – 16 Solutions

d) 3 only

Solution: b)
Learning: The simple majority system of representation does not
represent the whole electorate. On the other hand, even the smallest
section of the population gets its due share of representation in the
legislature in the PR system.

• There are two kinds of proportional representation, namely, single


transferable vote system and list system.

• In India, the first kind is adopted for the election of members to


the Rajya Sabha and state legislative council and for electing the
President and the Vice-President.

• Though some members of the Constituent Assembly had advocated


the system of proportional representation for the election of
members to the Lok Sabha, the Constitution has not adopted the
system due to two reasons.

o Difficulty for the voters to understand the system (which is


complicated) due to low literacy scale in the country.

o Unsuitability to the parliamentary government due to the


tendency of the system to multiply political parties leading to
instability in government.

Q Source: Chapter 22: Indian Polity: M Laxmikanth

86. Which of the following categories of information are


excluded from disclosure under the Right to Information Act,
2005?
1. Information received in confidence from foreign Government
2. Information which would impede the process of investigation of
offenders
www.insightsonindia.com
www.insightsias.com
TEST – 16 Solutions

3. Cabinet papers including records of deliberations of the Council


of Ministers

Select the correct answer using the codes below.

a) 1 and 2 only
b) 2 and 3 only
c) 1 and 3 only
d) 1, 2 and 3

Solution: d)
Learning: Other categories of information are:

• Information, disclosure of which would prejudicially affect the


sovereignty and integrity of India, the security, "strategic, scientific
or economic" interests of the State, relation with foreign State or
lead to incitement of an offense;
• Information which has been expressly forbidden to be published by
any court of law or tribunal or the disclosure of which may
constitute contempt of court;
• Information, the disclosure of which would cause a breach of
privilege of Parliament or the State Legislature;
• Information including commercial confidence, trade secrets or
intellectual property, the disclosure of which would harm the
competitive position of a third party, unless the competent
authority is satisfied that larger public interest warrants the
disclosure of such information.

Q Source: Chapter 8: India Yearbook 2016

87. Eurozone Crisis did NOT hit which of these countries?


1. Portugal
2. Spain
3. Italy
4. Ireland

Select the correct answer using the codes below.

a) 1 and 2 only
www.insightsonindia.com
www.insightsias.com
TEST – 16 Solutions

b) 3 and 4 only
c) 2 only
d) It hit all of them.

Solution: d)
Learning: The eurozone, a currency union of 17 European countries,
has been going through a major crisis which started with Greece but
spread rapidly to Ireland, Portugal, and Spain and subsequently Italy.

Sparked off by fear over the sovereign debt crisis in Greece, it went on to
impact the peripheral economies as well, especially those with over-
leveraged financial institutions. These economies, especially Greece,
have witnessed downgrades in the ratings of their sovereign debt due to
fears of default and a rise in borrowing costs.

Q Source: Chapter 10: Ramesh Singh: Indian Economy

88. Foreign exchange reserves are the foreign-currency deposits


held by national central banks and monetary authorities. Which of
these countries has the highest foreign exchange reserves?
a) Japan
b) China
c) USA
d) Russia

Solution: b)
Learning: China’s forex is approximately $3 trillion, Japan’s around 1.2
trillion. India’s forex is around $0.35 trillion.

China maintains a large reserve to manage its exchange rate by selling


and buying foreign currency in relation to the Chinese currency Yuan.

Q Source: Chapter 10: Ramesh Singh: Indian Economy


www.insightsonindia.com
www.insightsias.com
TEST – 16 Solutions

89. Consider the following statements.

Assertion (A): India is characterized by a relatively lower energy


intensity of GDP as compared to emerging economies like China and
South Africa.

Reason (R): Around 80 per cent of the crude oil consumed in India is
imported.

In the context of the above, which of these is correct?

a) A is correct, and R is an appropriate explanation of A.


b) A is correct, but R is not an appropriate explanation of A.
c) A is correct, but R is incorrect.
d) Both A and R are incorrect.

Solution: b)
Justification: It is lower as compared to China, South Africa, and
Russia but higher than that of Brazil. So, A is correct.

R is also correct, but it does not explain statement A because:

• Energy intensity of GDP is a structural characteristics of the


economy. The economy may import all its energy needs and yet
have higher energy intensity.
• Crude oil is just one part of the energy basket.

Q Source: Chapter 10: Ramesh Singh: Indian Economy

90. Consider the following statements about a money bill.


1. A money bill can only be introduced in the Lok Sabha.
2. Rajya Sabha cannot reject or amend a money bill.

Which of the above is/are correct?

a) 1 only
b) 2 only
c) Both 1 and 2
d) None

Solution: c)
www.insightsonindia.com
www.insightsias.com
TEST – 16 Solutions

Justification: The Constitution lays down a special procedure for the


passing of money bills in the Parliament.

A money bill can only be introduced in the Lok Sabha and that too on the
recommendation of the president. Every such bill is considered to be a
government bill and can be introduced only by a minister.

After a money bill is passed by the Lok Sabha, it is transmitted to the


Rajya Sabha for its consideration.

The Rajya Sabha has restricted powers with regard to a money bill. It
cannot reject or amend a money bill. It can only make the
recommendations.

It must return the bill to the Lok Sabha within 14 days, wither with or
without recommendations. The Lok Sabha can either accept or reject all
or any of the recommendations of the Rajya Sabha.

Q Source: Chapter 22: Indian Polity: M Laxmikanth

91.The Indian President, head of the State, is not elected directly by


the people of India. He is elected by members of an electoral
college consisting of
1. Former Presidents
2. Bharat Ratna Awardees
3. All Members of Parliament (MPs)
4. All Members of Legislative Assemblies of all states (MLAs)
5. All Members of Legislative Councils of states where applicable
(MLCs)

Select the correct answer using the codes below.

a) 3 and 4 only
b) 1, 2 and 3 only
c) 3, 4 and 5 only
d) None of the above
www.insightsonindia.com
www.insightsias.com
TEST – 16 Solutions

Solution: d)
Justification: Statement 1 and 2: They are not included in the
collegium.

Statement 3 and 4: Only elected members participate. Nominated


members do not. The rationale is that nominated members are chosen
by the incumbent President. If they participate in the election of the
President, it confers an unfair advantage to the incumbent.

Statement 5: MLCs do not participate in the election.

The elected members of the legislative assemblies of the Union


Territories of Delhi and Puducherry also participate. Its nominated
members do not participate.

Q Source: Chapter 17: Indian Polity: M Laxmikanth

92. The Adjournment motion is introduced in the Parliament to


draw attention of the House to a definite matter of urgent public
importance. Which of the following statements about it is
INCORRECT?
a) It needs the support of 50 members to be admitted.
b) It involves an element of censure against the government.
c) Rajya Sabha is not permitted to make use of this device.
d) If it is passed by a special majority, the sitting of the house is
adjourned after the discussion.

Solution: d)
Justification: As it interrupts the normal business of the House, it is
regarded as an extraordinary device.

It involves an element of censure against the government and hence


Rajya Sabha is not permitted to make use of this device. The discussion
on an adjournment motion should last for not less than two hours and
thirty minutes. So, (b) and (c) are correct.
www.insightsonindia.com
www.insightsias.com
TEST – 16 Solutions

The house need not be adjourned. So, (d) is wrong.

Q Source: Chapter 22: Indian Polity: M Laxmikanth

93. Which of the following is/are part of the Organised Indian


Money Market?
1. Mutual Funds
2. EXIM Bank
3. NABARD

Select the correct answer using the codes below.

a) 1 only
b) 1 and 3 only
c) 1 and 2 only
d) 2 and 3 only

Solution: a)
Learning: There are around eight instruments or components of the
Indian money market especially designed to fulfill the short-term fund
requirements of the different categories of the individuals, institutions or
the firms and companies:

• Treasury Bills
• Call Money Market
• Certificate of Deposit
• Commercial Bills
• Commercial Papers
• Mutual Funds
• Repo and Reverse Repo Markets.
• Cash Management Bill

NABARD and EXIM bank are part of the financial markets.

Q Source: Chapter 11: Ramesh Singh: Indian Economy


www.insightsonindia.com
www.insightsias.com
TEST – 16 Solutions

94. Which of the following is/are Financial Regulators in India?


1. National Housing Board
2. SIDBI
3. NABARD

Select the correct answer using the codes below.

a) 3 only
b) 1 and 3 only
c) 1 and 2 only
d) 1, 2 and 3

Solution: d)
Learning: NABARD finances and regulates the micro-finance sector in
India.

National Bank for Agriculture and Rural Development (NABARD), Small


Industries Development Bank of India (SIDBI), and National Housing
Bank (NHB) perform quasi-regulatory functions.

NABARD supervises regional rural banks as well as state and district


cooperative banks.

NHB regulates housing finance companies, and SIDBI regulates the state
finance corporations.

Q Source: Chapter 11: Ramesh Singh: Indian Economy

95. The President appoints a member of the Lok Sabha as the


Speaker Pro Tem when the Speaker of the last Lok Sabha vacates
her office. What are the duties of Speaker Pro Tem.
1. Administering oath of newly elected members
2. Enabling the house to select new Speaker

Which of the above is/are correct?


www.insightsonindia.com
www.insightsias.com
TEST – 16 Solutions

a) 1 only
b) 2 only
c) Both 1 and 2
d) None

Solution: c)
Learning: As provided by the Constitution, the Speaker of the last Lok
Sabha vacates his office immediately before the first meeting of the
newly- elected Lok Sabha.

Therefore, the President appoints a member of the Lok Sabha as the


Speaker Pro Tem.

• Usually, the seniormost member is selected for this.


• The President himself administers oath to the Speaker Pro Tem.
• The Speaker Pro Tem has all the powers of the Speaker. He
presides over the first sitting of the newly-elected Lok Sabha.
• His main duty is to administer oath to the new members.
• He also enables the House to elect the new Speaker.
• When the new Speaker is elected by the House, the office of the
Speaker Pro Tem ceases to exist. Hence, this office is a temporary
office, existing for a few days

Q Source: Chapter 22: Indian Polity: M Laxmikanth

96. Under the Liquidity Adjustment Facility (LAF)


a) RBI stands ready to lend to or borrow money from the
banking system, as per the latter’s requirement at fixed
interest rates
b) RBI manages the liquidity in the banking system by way of
sterilization
c) RBI sells government securities in the market to reduce the
money supply
www.insightsonindia.com
www.insightsias.com
TEST – 16 Solutions

d) RBI obliges the bank to categorise and prioritise their


lending limits to different sectors

Solution: a)
Learning: The LAF is the key element in the monetary policy operating
framework of the RBI (introduced in 2000).

On daily basis, the RBI stands ready to lend to or borrow money from
the banking system, as per the latter’s requirement, at fixed interest
rates.

The primary aim of such an operation is to assist banks to adjust to their


day-to-day mismatches in liquidity, via repo and reverse repo
operations.

OMOs are conducted by the RBI via the sale/purchase of government


securities (G-Sec) to/from the market, which is option (c).

Q Source: Chapter 11: Ramesh Singh: Indian Economy

97. The Prime Minister is the leader of the Lower House. In this
capacity, he enjoys which of the following powers?
1. He advises the President with regard to summoning and
proroguing of the sessions of the Parliament.
2. He can recommend dissolution of the Lok Sabha to President at
any time.
3. He has the authority to adjourn the lower house sine die.

Select the correct answer using the codes below.

a) 1 and 2 only
b) 2 and 3 only
c) 1 and 3 only
d) 1, 2 and 3

Solution: a)
www.insightsonindia.com
www.insightsias.com
TEST – 16 Solutions

Justification: Statement 2: Both the executive and legislature in India


have powers to shake each other’s existence. Executive (PM) can
recommend dissolution of lower house to the President.

On the other hand, legislature can pass no-confidence motion to remove


the council of Ministers from office.

Statement 3: Speaker, Lok Sabha only is authorized to do so.

Q Source: Chapter 19: Indian Polity: M Laxmikanth

98. Non-Banking financial companies (NBFCs) differ from banks


in
1. An NBFC cannot run a savings account for an individual, unlike
banks.
2. It cannot issue cheques to its customers, unlike banks.
3. No NBFC needs to get registered with RBI, unlike banks.

Select the correct answer using the codes below.

a) 1 and 2 only
b) 2 and 3 only
c) 1 and 3 only
d) 1, 2 and 3

Solution: a)
Justification: Statement 1: An NBFC cannot accept demand deposits
(which are payable on demand), like the savings and current accounts.

Statement 2: It is not a part of the payment and settlement system and


as such cannot issue cheques to its customers.

Statement 3: Under the RBI Act, 1934, the NBFCs have to get registered
with RBI. However, to obviate dual regulation, certain category of
NBFCs which are regulated by other regulators are exempted from the
requirement of registration with RBI.
www.insightsonindia.com
www.insightsias.com
TEST – 16 Solutions

Q Source: Chapter 11: Ramesh Singh: Indian Economy

99. Consider the following about the Indian Home Rule


movement.
1. It wanted to obtain the status of a Dominion for India within the
British Empire.
2. It worked as a joint movement of the Indian National Congress
and the All India Muslim League.
3. Gandhiji was once elected as a President of the All India Home
Rule League.

Select the correct answer using the codes below.

a) 1 and 2 only
b) 2 and 3 only
c) 1 and 3 only
d) 1, 2 and 3

Solution: c)
Justification: Statement 1: Between 1916 and 1918, when the World
War-I was gradually approaching an end, prominent Indians like
Muhammad Ali Jinnah, Bal Gangadhar Tilak, Sir S. Subramania Iyer,
Annie Besant decided to organise a national alliance of leagues across
India.

It was specifically to demand Home Rule, or self-government within the


British Empire for all of India.

Statement 2: It was run separately from both parties. Some members of


both INC and AIML joined the Home Rule movement.

Statement 3: In 1920, the All India Home Rule League elected Mahatma
Gandhi as its President. In a year, the body would merge into the Indian
National Congress to form a united Indian political front.
www.insightsonindia.com
www.insightsias.com
TEST – 16 Solutions

Q Source: Chapter 13: Bipin Chandra: India’s Struggle for


Independence

100. Which of the following is/are the executive powers and


functions of the Governor?
1. All executive actions of the government of a state are formally
taken in his name.
2. He can make rules for more convenient transaction of the
business of a state government and for the allocation among the
ministers of the said business.
3. He appoints the chief minister and other ministers.
4. He has the power to both appoint and remove the state election
commissioner.
5. He acts as the chancellor of universities in the state.
6. He can impose Governor’s rule in the State based on a report of
State Secretariat.

Select the correct answer using the codes below.

a) 1, 2, 3, 4 and 5 only
b) 1, 2, 3 and 4 and 6 only
c) 4, 5 and 6 only
d) 1, 2, 3 and 5 only

Solution: d)
Justification: Statement 1: He can make rules specifying the manner
in which the Orders and other instruments made and executed in his
name shall be authenticated.

Statement 2 and 3: He not only allocated business, but also appoints the
chief minister and other ministers.

Statement 4: He appoints the state election commissioner and


determines his conditions of service and tenure of office. However, the
www.insightsonindia.com
www.insightsias.com
TEST – 16 Solutions

state election commissioner can be removed only in like manner and on


the like grounds as a judge of a high court, and not by the Governor.

Statement 5: He also appoints the vice-chancellors of universities in the


state.

Statement 6: In the state of Jammu and Kashmir, failure of


constitutional machinery results in Governor's rule, imposed by invoking
Section 92 of Constitution of Jammu and Kashmir.

But, a) the proclamation can be issued by the state's Governor after


obtaining the consent of the President of India; and b) Governor’s
rule is not applicable to Indian states except J&K. So, this statement
would be incorrect.

Q Source: Chapter 26: Indian Polity: M Laxmikanth

INSIGHTS ON INDIA MOCK PRELIMINARY EXAM - 2016


INSIGHTS ON INDIA MOCK TEST – 16

GENERAL STUDIES

PAPER-I
Time Allowed: 2 Hours Maximum Marks: 200

INSTRUCTIONS
1. IMMEDITELY AFTER THE COMMENCEMENT OF THE EXAMINATION, YOU SHOULD
CHECK THAT THIS TEST BOOKLET DOES NOT HAVE ANY UNPRINTED OR TORN OR MISSING
PAGES OR ITEMS, ETC. IF SO, GET IT REPLACED BY A COMPLETE TEST BOOKLET.
2. You have to enter your Roll Number on the Test
Booklet in the Box provided alongside. DO NOT
Write anything else on the Test Booklet.
4. This Test Booklet contains 100 items (questions). Each item is printed only in English. Each item
comprises four responses (answers). You will select the response which you want to mark on the
Answer Sheet. In case you feel that there is more than one correct response, mark the response which
you consider the best. In any case, choose ONLY ONE response for each item.
5. You have to mark all your responses ONLY on the separate Answer Sheet provided. See directions in
the Answer Sheet.
6. All items carry equal marks.
7. Before you proceed to mark in the Answer Sheet the response to various items in the Test Booklet, you
have to fill in some particulars in the Answer Sheet as per instructions sent to you with your Admission
Certificate.
8. After you have completed filling in all your responses on the Answer Sheet and the examination has
concluded, you should hand over to the Invigilator only the Answer Sheet. You are permitted to take
away with you the Test Booklet.
9. Sheets for rough work are appended in the Test Booklet at the end.
10. Penalty for wrong answers :
THERE WILL BE PENALTY FOR WRONG ANSWERS MARKED BY A CANDIDATE IN THE
OBJECTIVE TYPE QUESTION PAPERS.
(i) There are four alternatives for the answer to every question. For each question for which a
wrong answer has been given by the candidate, one-third of the marks assigned to that
question will be deducted as penalty.
(ii) If a candidate gives more than one answer, it will be treated as a wrong answer even if one of
the given answers happens to be correct and there will be same penalty as above to that
question.
(iii) If a question is left blank, i.e., no answer is given by the candidate, there will be no penalty for
that question.

http://www.insightsonindia.com

INSIGHTS ON INDIA MOCK TEST SERIES FOR CIVIL SERVICES PRELIMINARY EXAM 2016

http://www.insightsonindia.com INSIGHTS Page 1






1. Consider the following statements 2. He causes to be laid before the
about the All-India trends of inflation. Parliament the Union Budget.
3. No demand for a grant can be made
Assertion (A): Urban inflation has always except on his recommendation.
been more than rural Inflation in post- 4. He can make advances out of the
independent India. contingency fund of India to meet
Reason (R): Urban India constitutes a larger any unforeseen expenditure.
share of India’s GDP than rural areas. 5. He constitutes the Union finance
commission to recommend the
In the context of the above, which of these is distribution of revenues between
correct? the Centre and the states.

a) A is correct, and R is an appropriate Select the correct answer using the codes
explanation of A. below.
b) A is correct, but R is not an
appropriate explanation of A. a) 3 and 5 only
c) A is correct, but R is incorrect. b) 1, 2 and 5 only
d) A is incorrect, but R is correct. c) 1, 2 and 4 only
d) 1, 2, 3, 4 and 5

2. Remittances are an important source


of financial flows for India. Which of 4. Ahmadabad Mill Strike 1918 led by
the following factors may lead to an Gandhiji was based on the issue of
increase in remittance inflows to a) Plague Bonus of the previous year
India? to workers
1. Higher oil prices in the Gulf b) Inhuman treatment of mill workers
countries by the management
2. Depreciation of Rupee c) Objection of management on
participation of workers in the
Which of the above is/are correct? freedom struggle
d) Large-scale layoffs of mill workers
a) 1 only
b) 2 only
c) Both 1 and 2
d) None 5. Hannover Messe 2015 was a/an
a) Energy summit held under the
aegis of Conference of Parties
(COP) of UNFCCC
3. Which of the following is/are the
b) The cultural exchange programme
financial powers of the President of
between India and Europe
India?
organized by the Ministry of
1. Money bills can be introduced in
Culture and Ministry of Tourism
the Parliament only with his prior
c) An annual global exhibition held in
recommendation.
Germany which provided a

http://www.insightsonindia.com INSIGHTS Page 2






platform for India to gain entry to 8. In case of a Parliamentary deadlock,
international markets the President summons a joint sitting
d) None of the above of both houses to resolve the issue.
Who chairs the joint sitting?
a) President
6. When a vacancy occurs in the office of b) Chairman, Rajya Sabha
the President due to his resignation, c) Speaker, Lok Sabha
the Vice-President acts as the d) Leader of the House of People
President until a new President is
elected. Consider the following with
regard to when the office of Vice- 9. Consider the following statements.
President is vacant.
Assertion (A): All doubts and disputes in
Assertion (A): Speaker, Lok Sabha acts as the connection with election of the President are
President of India in such a case. inquired into and decided by the Election
Commission of India.
Reason (R): Speaker, Lok Sabha is ranked
immediately below the Vice-President and Reason (R): Election Commission of India is
Prime Minister in the Table of Precedence. entrusted with the responsibility of
conducting the Presidential elections.
In the context of the above, which of these is
correct? In the context of the above, which of these is
correct?
a) A is correct, and R is an appropriate
explanation of A. a) A is correct, and R is an appropriate
b) A is correct, but R is not an explanation of A.
appropriate explanation of A. b) A is correct, but R is not an
c) A is incorrect, but R is correct. appropriate explanation of A.
d) Both A and R are incorrect. c) A is correct, but R is incorrect.
d) A is incorrect, but R is correct.

7. Which of the following statements


about the Archaeological Survey of 10. It is said that the US-India Defense
India (ASI) is INCORRECT? Technology and Partnership Act that
a) It was established before proposes to amend the ‘Arms Export
independence. Control Action’ is of immense strategic
b) It functions as an attached office of significance to India. What can be the
the Department of Culture. possible reason(s)?
c) ‘Expeditions abroad’ is one of the 1. It seeks to formalise India’s status
major activities of the organization. as a major partner of equal status
d) ASI cannot take up conservation as US’s NATO allies and closest
projects of sites outside India. partners.

http://www.insightsonindia.com INSIGHTS Page 3






2. It will elevate India's status in d) None
export of defence articles from the
United States to India.

Which of the above is/are correct? 13. In his oath of secrecy, the Prime
Minister swears that
a) 1 only 1. All government proceedings will be
b) 2 only kept secret.
c) Both 1 and 2 2. No matter handled by the Prime
d) None Minister’s Office will be disclosed
to other ministries.

Which of the above is/are correct?


11. The Ghadar party was a revolutionary
organization with the aim of securing a) 1 only
India's independence from British rule. b) 2 only
Who among the following were c) Both 1 and 2
associated with it? d) None
1. Lala Har Dayal
2. Rashbehari Bose
3. Sohan Singh Bhakna 14. Consider the following about Earth
Select the correct answer using the codes Hour.
below. 1. It is organized by the World Wide
Fund for Nature (WWF).
a) 1 and 2 only 2. The day organizes and encourages
b) 2 and 3 only planting of at least one tree by
c) 1 and 3 only every human being at a specific
d) 1, 2 and 3 time.
3. For India, the country-specific
theme for 2016 was to encourage
12. Consider the following statements. usage of solar energy.
1. The Constitution does not contain Select the correct answer using the codes
any specific procedure for the below.
selection and appointment of the
Prime Minister. a) 1 and 2 only
2. The Constitution does not require b) 2 and 3 only
that a person must prove his c) 1 and 3 only
majority in the Lok Sabha before he d) 1, 2 and 3
is appointed as the Prime Minister.

Which of the above is/are correct?

a) 1 only
b) 2 only
c) Both 1 and 2
http://www.insightsonindia.com INSIGHTS Page 4




15. Exchange-Traded Fund (ETF) is and advice of the council of
a) A basket of blue-chip company ministers, thus it helps him.
stocks
b) A basket of assets traded on stock
market 18. Consider the following about Cabinet
c) FII that is received in India via the Committees.
security markets 1. They are extra-constitutional
d) FDI in stock market companies bodies.
operating in India 2. They are only temporary in nature.
3. Non-cabinet ministers cannot be its
members.
16. Consider the following statements. 4. Only the Prime Minister is
1. The President may require the authorized to chair cabinet
Council of Ministers to reconsider committees.
advice tendered by it.
2. The advice tendered by Ministers to Select the correct answer using the codes
the President shall not be inquired below.
into in any court. a) 1 and 2 only
Which of the above is/are correct? b) 3 and 4 only
c) 1 only
a) 1 only d) 2 and 4 only
b) 2 only
c) Both 1 and 2
d) None 19. World Network of Biosphere Reserves
includes which of the following
protected areas of India?
17. In 1971, the Supreme Court held that 1. Gulf of Mannar
even after the dissolution of the Lok 2. Sundarban
Sabha, the council of ministers does 3. Namdapha
not immediately cease to hold office. 4. Kanchenjunga
How does it help the political system 5. Agasthyamala
and administration in India?
a) It prevents the bureaucracy from Select the correct answer using the codes
usurping political power. below.
b) It ensures the accountability of the a) 1, 2 and 5 only
executive to the legislature. b) 2, 3 and 4 only
c) The Council of Ministers get an c) 1, 3 and 5 only
opportunity to again prove their d) 1, 2, 3, 4 and 5
party majority on the house floor.
d) The president cannot exercise the
executive power without the aid

http://www.insightsonindia.com INSIGHTS Page 5






20. Consider the following statements. means of the single transferable
vote.
Assertion (A): Though the President of India 3. The seats are allotted to the states
is not a member of either House of in the Rajya Sabha on the basis of
Parliament, he is an integral part of the population.
institution of Parliament.
Select the correct answer using the codes
Reason (R): A bill passed by both the Houses below.
of Parliament cannot become law without the
President’s assent. a) 1 and 2 only
b) 2 and 3 only
In the context of the above, which of these is c) 1 and 3 only
correct? d) 1, 2 and 3 only
a) A is correct, and R is an appropriate
explanation of A.
b) A is correct, but R is not an 23. The Union Cabinet has given its
appropriate explanation of A. approval for India to accede to the
c) A is correct, but R is incorrect. Ashgabat Agreement. It establishes
d) A is incorrect, but R is correct. international transport and transit
corridor between
a) Central Asia and the Persian Gulf
21. The parliamentary form of government countries
emphasises on the b) South-east Asia to European
a) Separation of powers between the Countries
legislature and executive c) West-African countries to South
b) System of Universal Adult Asian countries
Franchise d) Oil producing Gulf countries and
c) Interdependence between the littoral states of Indian Ocean
legislative and executive organs
d) Democratic election system
administered by an independent 24. The Constitution has empowered the
body Parliament to prescribe the manner of
choosing the representatives of the
union territories in the Lok Sabha.
22. Consider the following statements Members of Lok Sabha from the Union
about Rajya Sabha. Territories are
1. The representatives of states in the a) Chosen by direct election
Rajya Sabha are elected by the b) Nominated by the
elected members of state legislative Administrator/Lieutenant
assemblies. Governor of the UT
2. The election to Rajya Sabha is held c) Recommended by the Union
in accordance with the system of Cabinet and chosen by the
proportional representation by President

http://www.insightsonindia.com INSIGHTS Page 6






d) Chosen by indirect elections by the 2. The retiring members of Rajya
local bodies of the UTs Sabha are eligible for re-election
and re-nomination any number of
times.
25. Consider the following about the 3. The Constitution fixes the term of
Delimitation Commission of India. the members of the Rajya Sabha.
1. It is a statutory body. 4. The President can curtail the term
2. It redraws the boundaries of both of a Rajya Sabha member any time
assembly and Lok Sabha based on the recommendation of
constituencies. the Union cabinet.
3. Its orders cannot be challenged in a Select the correct answer using the codes
court of law. below.
4. The Lok Sabha cannot modify its
orders. a) 2, 3 and 4 only
5. It is setup every five years. b) 1 and 4 only
c) 1 and 3 only
Select the correct answer using the codes d) 1 and 2 only
below.

a) 1, 2 and 5 only
b) 3, 4 and 5 only 28. Which of the following is/are
c) 1, 2, 3 and 4 only recognized as classical dance forms in
d) 1, 4 and 5 only India?
1. Odissi
2. Manipuri
26. Sevottam is related to 3. Sattriya
a) Citizen’s charter in public 4. Mohiniyattam
organizations Select the correct answer using the codes
b) Grievance redressal programmes below.
within the most vulnerable
communities a) 2, 3 and 4 only
c) Reform of Public Sector Banks b) 1, 2 and 3 only
(PSBs) c) 3 and 4 only
d) Providing support to first d) 1, 2, 3 and 4
generation entrepreneurs under
‘Make in India’ programmes
29. The governor can reserve a bill passed
by the State Legislature for the
27. Consider the following about Rajya consideration of the President in which
Sabha. of the following cases?
1. It is a permanent body and not 1. If it is against the provisions of the
subject to dissolution. Constitution.

http://www.insightsonindia.com INSIGHTS Page 7






2. If it is opposed to the Directive c) 1, 2 and 3 only
Principles of State Policy. d) 1 and 4 only
3. If it is against the larger interest of
the country or of grave national
importance. 31. The institutions of Speaker and Deputy
4. If it endangers the position of the Speaker originated in India in 1921
State High Court under
Select the correct answer using the codes a) A Charter issued by the British
below. crown for improving legislative
business in British India
a) 1 and 4 only b) Provisions of the Government of
b) 1, 2 and 3 only India Act of 1919 (Montague–
c) 2, 3 and 4 only Chelmsford Reforms)
d) 1, 2, 3 and 4 c) An order issued by the then
Governor-General of India to
manage the Central Legislative
30. The Speaker of the Lok Sabha derives Assembly better
his powers and duties from the d) Amendments made to the Charter
Constitution of India, the Rules of Act of 1891 that created the office of
Procedure and Conduct of Business of President and Vice-President of
Lok Sabha, and Parliamentary Legislative assemblies
Conventions. Accordingly, she
exercises which of the following
powers? 32. In reverse repos
1. She is the final interpreter of the a) The banks and financial institutions
provisions of the Constitution of purchase Government securities
India for any matter relating to the from the RBI
Parliament. b) Banks and the financial institutions
2. She has the final power in matters borrow money from the RBI for the
of decorum, order and business in short-term
the house. c) RBI buys commercial papers from
3. She decides whether a bill is a the public
money bill or not. d) None of the above
4. She appoints the chairman of all
the parliamentary committees of
the Lok Sabha and supervises their 33. Each House of Parliament has separate
functioning. secretarial staff of its own. The
Select the correct answer using the codes secretariat of each House is headed by
below. a secretary-general. He is a permanent
officer and is appointed by the
a) 2 and 3 only a) Leader of the House
b) 2, 3 and 4 only b) Presiding Officer of the House

http://www.insightsonindia.com INSIGHTS Page 8






c) President of India b) 2 and 3 only
d) The Department of Personnel & c) 1 and 3 only
Training d) 1, 2 and 3

34. Which of the following correctly point 36. A starred question in Parliament,
out the difference(s) between distinguished by an asterisk
Adjournment and Prorogation of a a) Requires an oral answer and hence
house? supplementary questions can follow
1. Unlike Prorogation, Adjournment b) Requires a written answer and
only terminates a sitting and not a hence supplementary questions can
session of the House. follow
2. Unlike Adjournment, Prorogation c) Any question to which Ministers
leads to lapse of all the bills or any alone are answerable
other business pending before the d) One that is asked by giving a notice
House. of less than ten days and answered
orally
Select the correct answer using the codes
below.

a) 1 only 37. Which of the following statements


b) 2 only about the Zero Hour in Parliament is
c) Both 1 and 2 INCORRECT?
d) None a) The zero hour is not mentioned in
the Rules of Procedure of the
house.
35. When the Lok Sabha is dissolved, all b) Matters can be raised by members
business including bills, motions, without prior notice.
resolutions, notices, petitions and so c) The zero hour starts before the
on pending before it or its committees question hour after which the
lapse. However, certain bills do not agenda for the day is taken up.
lapse on the dissolution of the Lok d) It is an Indian innovation in the
Sabha. They are? field of parliamentary procedures
1. A bill passed by the Lok Sabha but and has been in existence since
pending in the Rajya Sabha 1962.
2. A bill pending in the Rajya Sabha
but not passed by the Lok Sabha
3. A bill passed by both Houses but 38. Which of the following correctly point
pending assent of the president out the difference(s) between Censure
Motion and No Confidence Motion?
Select the correct answer using the codes 1. No-confidence motion should state
below. the reasons for its adoption in the
a) 1 and 2 only

http://www.insightsonindia.com INSIGHTS Page 9






Lok Sabha, Censure motion need
not.
2. No-confidence motion can only be 41. Article 110 of the Constitution deals
moved against the entire council of with the definition of money bills. It
ministers, Censure motion can be states that a bill is deemed to be a
moved only against individual money bill if it contains ‘only’
ministers. provisions dealing with all or any of
which of the following matters?
Which of the above is/are correct? 1. Abolition of a tax
2. Regulation of the borrowing of
a) 1 only money by the Union government
b) 2 only 3. Appropriation of money out of the
c) Both 1 and 2 Consolidated Fund of India
d) None
Select the correct answer using the codes
below.
39. The Financial Stability Board (FSB) a) 1 and 2 only
that promotes international financial b) 2 and 3 only
stability through enhanced c) 1 and 3 only
information exchange and d) 1, 2 and 3
international cooperation in financial
market supervision was established by
a) The G-8 finance ministers and
central bank governors 42. Advanced countries, in particular EU
b) G-20 major economies countries and Japan, have been
c) United Nations Economic and witnessing a decline in the energy
Social Council (UNECOSOC) intensity of GDP due to which of the
d) A resolution of all members of the following factors possibly?
UN General Assembly 1. Technological improvement
2. Shift in the structure of their
economies towards services

40. Which of the following statements Which of the above is/are correct?
about Public Bills is INCORRECT?
1. It is introduced in the Parliament a) 1 only
by a minister. b) 2 only
2. It can be introduced without any c) Both 1 and 2
prior notice in the house. d) None

Which of the above is/are correct?

a) 1 only
b) 2 only
c) Both 1 and 2
d) None

http://www.insightsonindia.com INSIGHTS Page 10






43. A joint sitting to resolve the 5. Passing of appropriation bill.
Parliamentary deadlock applies to 6. Voting on demands for grants.
which of the following types of bills
apart from ordinary bills? Select the correct answer using the codes
1. Appropriation Bill as a part of below.
Annual Budget a) 132564
2. Constitutional Amendment Bills b) 312654
3. Financial Bills c) 312456
Select the correct answer using the codes d) 132654
below.

a) 1 and 2 only 46. ‘Vote on account’ provision is related


b) 2 and 3 only to
c) 3 only a) Any grant made in advance by the
d) 1 only Lok Sabha to the executive before
the passing of appropriation bill
b) Excess grant awarded by the
44. The expenditure ‘charged’ upon the Parliament to the executive that
Consolidated Fund of India had been appropriated in the
1. Cannot be discussed by Parliament annual budget
2. Is non-votable by the Rajya Sabha, c) Grant made before the ‘March
but votable by Lok Sabha Rush’ by the Lok Sabha to the
3. Does not require Presidential executive
assent d) Grant sanctioned by Parliament to
executive for meeting an
Select the correct answer using the codes unexpected demand upon the
below. resources of India
a) 1 and 2 only
b) 1 only
c) 2 and 3 only 47. Which of the following statements with
d) None of the above reference to the Public Account of
India is CORRECT?
a) It is operated by executive action.
45. The budget goes through six stages in b) All taxes collected by the
the Parliament. Arrange the following Government of India are submitted
in the correct order of proceeding in to this account.
Parliament. c) It is used to fund contingencies and
1. Presentation of budget. disaster management.
2. Scrutiny by departmental d) It is an extra-constitutional fund.
committees.
3. General discussion.
4. Passing of finance bill.

http://www.insightsonindia.com INSIGHTS Page 11






48. Which of the following is the authority
to create or abolish the state legislative 51. Consider the following statements.
councils?
a) President of India Assertion (A): The single system of courts
b) Union Council of Ministers that enforces both Central laws as well as the
c) Parliament state laws has been adopted in India from the
d) Governor of the Concerned State Government of India Act of 1935.

Reason (R): The Government of India Act of


1935 established the Federal Court of India
49. In 2014, Supreme court of India setup with original, appellate and advisory
a Committee under NR Madhava jurisdiction.
Menon to frame guidelines for
a) Allowing new mining operations in In the context of the above, which of these is
left wing extremism affected areas correct?
b) Government advertisements in a) A is correct, and R can be an
print and media appropriate explanation of A.
c) Regulation of economic activities in b) A is correct, but R cannot be an
Ecologically Sensitive Zones (ESZs) appropriate explanation of A.
d) Improving the socio-economic c) A is correct, but R is incorrect.
status of the Transgender d) A is incorrect, but R is correct.

50. Consider the following statements 52. A dispute between two or more states
about a certain Parliamentary will fall under Supreme Court’s
Committee. a) Exclusive Original Jurisdiction
1. The Rajya Sabha has no b) Appellate Jurisdiction
representation in this committee. c) Writ Jurisdiction
2. The committee examine the d) Advisory Jurisdiction
estimates included in the budget
and suggest ‘economies’ in public
expenditure.
53. The Supreme Court enjoys appellate
3. The committee works throughout
jurisdiction in
the financial year and reports to the
1. Civil matters
House as its examination proceeds.
2. Criminal matters
The above refer to? 3. Constitutional matters

a) Committee on Public Undertakings Select the correct answer using the codes
b) Departmental Standing below.
Committees
a) 1 and 2 only
c) Committee on Government
b) 2 and 3 only
Finances
c) 1 and 3 only
d) Estimates Committee
d) 1, 2 and 3
http://www.insightsonindia.com INSIGHTS Page 12




2. The Constitution does not lay down
the grounds upon which a governor
54. The constitutional validity of a may be removed by the President.
legislative enactment or an executive
order can be challenged in the Which of the above is/are correct?
Supreme Court on which of the
following grounds? a) 1 only
1. If it infringes the Fundamental b) 2 only
Rights c) Both 1 and 2
2. If it is repugnant to the d) None
constitutional provisions
3. If it is outside the competence of
the authority which has framed it 57. Before entering upon his office, the
President has to make and subscribe to
Select the correct answer using the codes an oath or affirmation. Which of the
below. following is/are part of the oath or
a) 1 and 2 only affirmation made by the President?
b) 2 and 3 only 1. To preserve, protect and defend the
c) 1 and 3 only Constitution and the law
d) 1, 2 and 3 2. To uphold the sovereignty and
integrity of India
3. Perform the duties of the office
without fear or favour
55. How is the Governor of a state
appointed? Select the correct answer using the codes
a) Indirectly elected by the State below.
Legislative Assembly
b) Nominated by the President a) 1 and 2 only
c) Elected by a collegium consisting of b) 2 and 3 only
the heads of all local bodies of the c) 1 only
concerned State d) 1, 2 and 3
d) Nominated by the State
Government subject to the approval
of the Union Government 58. How is the Lok Sabha speaker chosen?
a) Nominated by a collegium of Prime
Minister, Leader of Opposition and
56. Consider the following about the terms Minister for Parliamentary affairs
of the Office of the Governor. b) Nominated by the party/coalition
1. The Governor can resign at any in majority in Lok Sabha
time by addressing a resignation c) Elected by the house from a body of
letter to the Chief Minister of the eminent citizens specially selected
State. for this purpose
d) Elected by the house from amongst
its members

http://www.insightsonindia.com INSIGHTS Page 13






61. The Union Commerce and Industry
Ministry has notified foreign direct
59. Which of the following is/are the investment (FDI) upto 49% in
differences between the pardoning insurance and pension sector will be
Powers of President and Governor? under automatic route. What
1. Governor cannot pardon a death implication it will have for the sectors?
sentence, President can. 1. The Indian corporate laws would
2. Governor can pardon a sentence of not apply to companies investing
court martial, President cannot. within the 49% FDI limit.
Which of the above is/are correct? 2. Foreign Companies investing in
these sectors within the cap would
a) 1 only not require prior approval from
b) 2 only government.
c) Both 1 and 2
d) None Which of the above is/are correct?

a) 1 only
b) 2 only
60. The Governor has constitutional c) Both 1 and 2
discretion in which of the following d) None
cases?
1. Recommendation for the
imposition of the President’s Rule 62. Consider the following about the
in the state ordinance-making Power of the
2. While exercising his functions as President.
the administrator of an adjoining 1. The ordinances have the same force
union territory and effect as an act of Parliament.
3. Seeking information from the chief 2. An ordinance can be issued even if
minister with regard to the only one House of Parliament is in
administrative and legislative session.
matters of the state 3. It can be issued to amend the
4. Reservation of a bill for the Constitution.
consideration of the President. 4. It is subject to judicial review.
Select the correct answer using the codes Select the correct answer using the codes
below. below.
a) 1 and 2 only a) 1, 3 and 4 only
b) 3 and 4 only b) 1 only
c) 1, 3 and 4 only c) 1, 2 and 4 only
d) 1, 2, 3 and 4 d) 2 and 3 only

http://www.insightsonindia.com INSIGHTS Page 14






63. The president nominates 12 members 66. Consider the following statements
to the Rajya Sabha from about the role of the Prime Minister.
a) The persons recommended by the 1. He allocates and reshuffles various
National Integration Council portfolios among the ministers.
b) People who have special knowledge 2. He can bring about the collapse of
or practical experience in art, the council of ministers by
literature, science and social resigning from office.
service.
c) People who have contributed to Which of the above is/are correct?
immensely to Indian politics. a) 1 only
d) Eminent political scientists who b) 2 only
have never contested an election c) Both 1 and 2
d) None

64. Nepal has recently signed an


agreement with China to build a 67. The Union Ministry of Health and
strategic railway link between the two Family Welfare has recently amended
countries through Tibet. What the Schedule V of the Drugs and
implication does it have for India? Cosmetics Rules, 1945. As per the new
1. It may reduce Nepal’s dependency rules issued by the Ministry
on Indian sea ports. a) Clinical trials in India will be
2. It may reduce India’s dominance of banned.
Nepal’s overall trade with Asia. b) Prior government permission will
Which of the above is/are correct? be needed for clinical trials.
c) Clinical trials will be banned for
a) 1 only academic purposes.
b) 2 only d) None of the above
c) Both 1 and 2
d) None
68. Which of the following statements
about the Public Accounts Committee
65. Part V of the Indian constitution deals is INCORRECT?
with which of the following matters? a) Its members are nominated by the
a) Organisation, privileges and powers Speaker giving due representation
of the Parliament from all parties.
b) Special provisions relating to b) The term of office of the members
certain classes is one year.
c) Relations between the Union and c) A minister cannot be chosen as a
the States member of the committee.
d) Provisions for acquisition and d) The committee examines the
termination of citizenship annual audit reports of the

http://www.insightsonindia.com INSIGHTS Page 15






comptroller and auditor general of d) Hub of mutiny and revolutionary
India (CAG). thought in India

69. ‘Drishti’ deployed by the Indian 72. Kutiyattam is a form of


Metrological Department helps a) Folk art originating from the early
a) Provide real time visibility Tamil country
information to pilots b) Sanskrit theatre traditionally
b) Provide quick weather forecasts to performed in the state of Kerala
farmers c) Dance performed by female
c) Detect early cyclones and tsunamis members of the Luriyum
in coastal areas community of Andhra Pradesh
d) Track the buildup of Monsoon in d) Poetic performance accompanied
Northern region by a Mridangam in South India

70. To be eligible for election as Vice- 73. The question of disqualification under
President, a person should fulfil which the Anti-defection provisions under
of the following qualifications? Tenth Schedule of the constitution is
1. He should be qualified for election decided by the
as a member of the Rajya Sabha. a) Chairman in the case of Rajya
2. He should not be affiliated to a Sabha and Speaker in the case of
political party. Lok Sabha
3. He should not have ever held the b) Supreme Court in case of
office of the President. Parliament and concerned High
courts in case of State legislative
Select the correct answer using the codes assemblies
below. c) President of India in all cases based
a) 1 only on the recommendations of the
b) 2 and 3 only Union cabinet
c) 1 and 3 only d) Election Commission of India
d) 1 and 2 only

74. Which of the following fellowships are


71. “Yugantar Ashram” is best known as awarded by Sahitya Akademi?
the 1. Sahitya Akademi Honorary
a) First war memorial established by Fellowhip
the British in respect of Indian 2. Anand Fellowship
freedom fighters 3. Rabindranath Tagore Fellowship
b) Centre of Indian National Select the correct answer using the codes
Congress’s agitations below.
c) Headquarters of the Gadar party

http://www.insightsonindia.com INSIGHTS Page 16






a) 1 and 2 only d) None
b) 2 and 3 only
c) 1 and 3 only
d) 1, 2 and 3 77. Consider the following about the
trends of National Income in India.
1. India’s GDP growth rate has always
75. Consider the following about the increased post-2004.
powers of the President of India. 2. India’s GDP has never shrunk since
1. He can appoint an inter-state 1991 reforms.
council to promote Centre–state
and inter-state cooperation. Which of the above is/are correct?
2. He directly administers the union a) 1 only
territories through administrators b) 2 only
appointed by him. c) Both 1 and 2
3. He can declare an area as d) None
scheduled area and has powers
with respect to the administration
of scheduled areas and tribal areas.
78. Consider the following statements.
Select the correct answer using the codes
Assertion (A): India is the largest exporter in
below.
commercial services.
a) 1 and 2 only
Reason (R): Services sector contributes to
b) 2 and 3 only
more than 50% of India’s GDP.
c) 1 and 3 only
d) 1, 2 and 3 In the context of the above, which of these is
correct?

a) A is correct, and R is an appropriate


76. Consider the following about the India
explanation of A.
Tourism Development Corporation
b) A is correct, but R is not an
Limited (ITDC).
appropriate explanation of A.
1. It is owned by the Government of
c) A is correct, but R is incorrect.
India and functions under the
d) A is incorrect, but R is correct.
Ministry of Culture.
2. It can develop tourism
infrastructure through the public-
private partnership (PPP) mode in 79. Which of the following sectors is/are
the country. included in the Index of industrial
production (IIP)?
Which of the above is/are correct? 1. Mining
2. Electricity
a) 1 only
3. Fertilizers
b) 2 only
c) Both 1 and 2

http://www.insightsonindia.com INSIGHTS Page 17






Select the correct answer using the codes 82. The main objectives of the Special
below. Economic Zone (SEZ) Act, 2005 are
1. Promotion of exports of goods and
a) 1 and 2 only services
b) 2 and 3 only 2. Promotion of investment from
c) 1 and 3 only foreign sources
d) 1, 2 and 3 3. Development of infrastructure
facilities

80.The Union executive consists of the Select the correct answer using the codes
1. President below.
2. Vice-President a) 1 and 2 only
3. Prime Minister b) 2 and 3 only
4. Council of Ministers c) 1 and 3 only
5. Attorney-General of India d) 1, 2 and 3
6. Central Administrative Tribunal
7. Central Vigilance Commission

Select the correct answer using the codes 83. Consider the following about the
below. impeachment of President of India.
1. The impeachment charges can be
a) 1, 2, 3, 4 and 5 only initiated by either House of
b) 1, 2, 5, 6 and 7 only Parliament.
c) 3 and 4 only 2. The Parliament investigates the
d) 1, 2, 3, 4, 5, 6 and 7 impeachment charges.
3. No President has so far been
impeached.
81. India has signed a free trade
agreement (FTA) with which of these Select the correct answer using the codes
nations? below.
1. Sri Lanka a) 1 and 2 only
2. China b) 2 and 3 only
3. European Union c) 1 only
Select the correct answer using the codes d) 1, 2 and 3
below.

a) 1 and 2 only
b) 1 only
c) 1 and 3 only 84. Project Arrow related with Service
d) 3 only Quality Improvement is an initiative of
a) Department of Posts
b) Ministry of Defence
c) Ministry of Tourism

http://www.insightsonindia.com INSIGHTS Page 18






d) Ministry of Tribal Welfare

87. Eurozone Crisis did NOT hit which of


these countries?
85. The system of proportional 1. Portugal
representation aims at removing the 2. Spain
defects of territorial representation, 3. Italy
where all sections of the people get 4. Ireland
representation in proportion to their
number. What types of proportional Select the correct answer using the codes
representation are followed in India? below.
1. List system
2. Single transferable vote system a) 1 and 2 only
3. Mixed member system b) 3 and 4 only
c) 2 only
Select the correct answer using the codes d) It hit all of them.
below.

a) 1 and 2 only
b) 2 only 88. Foreign exchange reserves are the
c) 1 and 3 only foreign-currency deposits held by
d) 3 only national central banks and monetary
authorities. Which of these countries
has the highest foreign exchange
reserves?
86. Which of the following categories of a) Japan
information are excluded from b) China
disclosure under the Right to c) USA
Information Act, 2005? d) Russia
1. Information received in confidence
from foreign Government
2. Information which would impede
the process of investigation of 89. Consider the following statements.
offenders Assertion (A): India is characterized by a
3. Cabinet papers including records of relatively lower energy intensity of GDP as
deliberations of the Council of compared to emerging economies like China
Ministers and South Africa.
Select the correct answer using the codes Reason (R): Around 80 per cent of the crude
below. oil consumed in India is imported.
a) 1 and 2 only In the context of the above, which of these is
b) 2 and 3 only correct?
c) 1 and 3 only
d) 1, 2 and 3 a) A is correct, and R is an appropriate
explanation of A.

http://www.insightsonindia.com INSIGHTS Page 19






b) A is correct, but R is not an 92. The Adjournment motion is
appropriate explanation of A. introduced in the Parliament to draw
c) A is correct, but R is incorrect. attention of the House to a definite
d) Both A and R are incorrect. matter of urgent public importance.
Which of the following statements
about it is INCORRECT?
90. Consider the following statements a) It needs the support of 50 members
about a money bill. to be admitted.
1. A money bill can only be b) It involves an element of censure
introduced in the Lok Sabha. against the government.
2. Rajya Sabha cannot reject or c) Rajya Sabha is not permitted to
amend a money bill. make use of this device.
d) If it is passed by a special majority,
Which of the above is/are correct? the sitting of the house is adjourned
after the discussion.
a) 1 only
b) 2 only
c) Both 1 and 2
d) None 93. Which of the following is/are part of
the Organised Indian Money Market?
1. Mutual Funds
2. EXIM Bank
91. The Indian President, head of the
3. NABARD
State, is not elected directly by the
people of India. He is elected by Select the correct answer using the codes
members of an electoral college below.
consisting of
1. Former Presidents a) 1 only
2. Bharat Ratna Awardees b) 1 and 3 only
3. All Members of Parliament (MPs) c) 1 and 2 only
4. All Members of Legislative d) 2 and 3 only
Assemblies of all states (MLAs)
5. All Members of Legislative Councils
of states where applicable (MLCs) 94. Which of the following is/are Financial
Regulators in India?
Select the correct answer using the codes 1. National Housing Board
below. 2. SIDBI
3. NABARD
a) 3 and 4 only
b) 1, 2 and 3 only Select the correct answer using the codes
c) 3, 4 and 5 only below.
d) None of the above a) 3 only
b) 1 and 3 only
c) 1 and 2 only
d) 1, 2 and 3

http://www.insightsonindia.com INSIGHTS Page 20






2. He can recommend dissolution of
the Lok Sabha to President at any
95. The President appoints a member of time.
the Lok Sabha as the Speaker Pro Tem 3. He has the authority to adjourn the
when the Speaker of the last Lok Sabha lower house sine die.
vacates her office. What are the duties
of Speaker Pro Tem. Select the correct answer using the codes
1. Administering oath of newly below.
elected members
2. Enabling the house to select new a) 1 and 2 only
Speaker b) 2 and 3 only
c) 1 and 3 only
Which of the above is/are correct? d) 1, 2 and 3

a) 1 only
b) 2 only
c) Both 1 and 2 98. Non-Banking financial companies
d) None (NBFCs) differ from banks in
1. An NBFC cannot run a savings
account for an individual, unlike
banks.
96. Under the Liquidity Adjustment 2. It cannot issue cheques to its
Facility (LAF) customers, unlike banks.
a) RBI stands ready to lend to or 3. No NBFC needs to get registered
borrow money from the banking with RBI, unlike banks.
system, as per the latter’s
requirement at fixed interest rates Select the correct answer using the codes
b) RBI manages the liquidity in the below.
banking system by way of
sterilization a) 1 and 2 only
c) RBI sells government securities in b) 2 and 3 only
the market to reduce the money c) 1 and 3 only
supply d) 1, 2 and 3
d) RBI obliges the bank to categorise
and prioritise their lending limits to
different sectors 99. Consider the following about the
Indian Home Rule movement.
1. It wanted to obtain the status of a
97. The Prime Minister is the leader of the Dominion for India within the
Lower House. In this capacity, he British Empire.
enjoys which of the following powers? 2. It worked as a joint movement of
1. He advises the President with the Indian National Congress and
regard to summoning and the All India Muslim League.
proroguing of the sessions of the
Parliament.
http://www.insightsonindia.com INSIGHTS Page 21




3. Gandhiji was once elected as a
President of the All India Home
Rule League.

Select the correct answer using the codes


below.

a) 1 and 2 only
b) 2 and 3 only
c) 1 and 3 only
d) 1, 2 and 3

100. Which of the following is/are


the executive powers and functions of
the Governor?
1. All executive actions of the
government of a state are formally
taken in his name.
2. He can make rules for more
convenient transaction of the
business of a state government and
for the allocation among the
ministers of the said business.
3. He appoints the chief minister and
other ministers.
4. He has the power to both appoint
and remove the state election
commissioner.
5. He acts as the chancellor of
universities in the state.
6. He can impose Governor’s rule in
the State based on a report of State
Secretariat.

Select the correct answer using the codes


below.

a) 1, 2, 3, 4 and 5 only
b) 1, 2, 3 and 4 and 6 only
c) 4, 5 and 6 only
d) 1, 2, 3 and 5 only

http://www.insightsonindia.com INSIGHTS Page 22

Das könnte Ihnen auch gefallen